026267997
026267997
1. Apreviously well 27 year old woman presents with a history b. Central Venous Pressure Monitoring
of transient ischaemic attack affecting her right side and c. Chest XRay
speech. On examination there was nothing abnormal to find.
d. Echocardiography
An ECG, chest Xray,CT brain scan and routinehaematology
and biochemistry were all normal. What is the most likely e. Pulmonary Capillary Wedge Pressure Monitoring
underlying abnormality?
7. Whichof the following is a recognised feature of massive
a. atrialmyxoma
pulmonary embolism?
b. carotid artery stenosis
a. reduced plasma lactate levels
c. embolus from paroxysmal atrial fibrillation
b. an increase in serum troponin levels
d. patent foramen ovale
c. an arterial pH less than 7.2
e. subarachnoidhaemorrhage
d. blood gases show increased pCO2 on air
2. A 51 year old woman has had several syncopal episodes over e. normalDdimerlevels
the past year. Each episode is characterized by sudden but
brief loss of consciousness. She has no chest pain. She has no 8. A 60 year old man has worsening congestive heart failure with
ankle edema. On brain MRI there is a 1.5 cm cystic area in the increasing pulmonary oedema. His blood pressure is normal.
left parietal cortex. A chest Xray shows no cardiac enlargement, He has been healthy all his life with no major illnesses. A
and her lungfields are normal. Her serum total cholesterol is serum glucose is 5.6 mmol/L. His total serum cholesterol is
6.5 mmol/L. Which of the following cardiac lesions is shemost 4.8 mmol/L. The serum creatine kinase is not elevated.
likely to have? The most likely explanation for these findings is?
a. Cardiac amyloidosis a. Alcoholic cardiomyopathy
b. Ischemic cardiomyopathy b. Aortic dissection
c. Left atrial myxoma c. Calcified bicuspid aortic valve
d. Mitral valve prolapse d. Mitral valve annulus calcification
e. Tuberculous pericarditis e. Tricuspid valve endocarditis
3. A66yearoldman has developed chronic renal failure with a 9. Duringauscultation of the heart you discover a wide fixed
serum urea of 60 mmol/L and creatinine of 650 micromol/L. splitting of the second heart sound. In which of the following
Auscultation of the chest reveals a friction rub over the cardiac conditions does this occur?
apex. He is most likley to have a pericarditis that is termed? a. an uncomplicated ASD
a. Constrictive b. Fallot’s tetralogy
b. Fibrinous c. aortic stenosis
c. Hemorrhagic d. Right Bundle Branch Block
d. Purulent e. constrictive pericarditis
e. Serous
10. A 62 year old man has experienced substernal chest pain upon
4. WhichONE of the following is a contraindication to exertion with increasing frequency over thepast 1 year. An
thrombolysis? electrocardiogram shows T wave inversion in the anterolateral
leads at rest. He has a totalserum cholesterol of 7.0 mmol/l.
a. age over 75 years
On angiography, he has an 85% narrowing of the left anterior
b. the presence of atrial fibrillation descendingartery. Which of the following events is most
c. asthma likely to occur in this patient?
d. pregnancy a. A systemic artery embolus from thrombosis in a peripheral
e. background diabetic retinopathy vein.
b. A systemic artery embolus from a left atrial mural thrombus.
5. Whichof the following antimicrobialsis associated with c. Pulmonary embolism from a left ventricular mural
prolongation of the QT interval? thrombus
a. Coamoxiclav d. A systemic artery embolus from a left ventricular mural
b. Gentamicin thrombus.
c. Cefuroxime e. Pulmonary embolism from thrombosis in a peripheral vein.
d. Erythromycin
11. A17 year old girl was found collapsed and drowsy. Her
e. Isoniazid 12leadECG showed a sinus tachycardia of 120 beats per minute
with a corrected QT interval of 500 ms (normal <470). Which of
6. A60yearoldman presents with an inferior MI and receives the following is the most likely cause of her presentation?
thrombolysis. 4 hours following initial presentation he
becomes acutely breathless. His ECG demonstrates sinus a. Amphetamine
tachycardia (rate 108bpm) with T wave inversion inferiorly. b. Diphenhydramine
His ST segments are normal. On examination his JVP is c. Glue sniffing
elevated at 5 cm. Chest wasclear to auscultation. Following 80
d. Methadone
mg of Frusemide he eteriorates. His BP is now 80/60 and his
urine outputover the last 2 hours is 5 mls. What is the best e. Methanol
investigativeover the last 2 hours is 5 mls. What is the best
investigative measure?
a. Arterial Blood Gases
12. Arandomiseddoubleblindplacebo controlled study of a replaced. Which of the following pathologic findings in the 1117
cholesterollowering drug for the primary prevention of bioprosthesishas most likely led to the need for replacement?
coronary heart disease was conducted. It had a five year a. Calcification with stenosis
follow up period.The results showed an absolute risk of
myocardial infarction in the groupreceivingplacebo during b. Dehiscence
was 10 percent. The relative risk of those given the cholesterol c. Infective endocarditis
lowering medication was 0.8 What number of patientswill d. Strut failure
need to be treated with the drug for five years to prevent one
e. Thrombosis
myocardial infarction?
a. 20 18. Arandomised, doubleblind, placebo controlled trial of a
b. 40 cholesterol lowering drug in the primary prevention of
coronary heart disease is reported. 1000 subjects are treated
c. 50
with the active drug, and 1000 aregiven placebo. They are
d. 80 followed up over a five year period and 100 individuals in
e. 100 the placebo group and 80 inthe treatment group suffer a
CARDIOLOGY
myocardial infarction. What is the annual percentage risk of
13. Which ONE of the following is true regarding acute pulmonary myocardial infarction in the group treated with placebo?
embolism? a. 0.5%
a. a normal ECG excludes the diagnosis b. 2%
b. embolectomy is more effective than thrombolysis in c. 5%
improving survival
d. 8%
c. Heparin is as effective as thrombolytic therapy
e. 10%
d. the presence of hypoxaemia is an indication for thrombolysis
e. thrombolysis administered through a peripheral vein is as 19. A25yearoldpreviously healthy woman has worsening fatigue
effective as through a pulmonary artery catheter with dyspnoea, palpitations, and fever over the past one week.
Her vital signs on admission to the hospital show Temperature
14. A 70 yearoldwoman has a history of dyspnoea and palpitations 38.9°C Respiratory rate 30/min Pulse 105 bpm and BP 95/65
for six months. An ECG at that time showed atrial fibrillation. mmHg. Her heart rate is irregular. An ECG shows diffuse STT
She was given digoxin, diuretics and aspirin. She now segment changes. A Chest Xray shows mild cardiomegaly.
presents with two shortlived episodes of altered sensation in An echocardiogram shows slight mitral and tricuspid
the left face, left arm and leg. There is poor coordination of the regurgitation but no valvularvegetations. Her troponin I is 12
left hand. ECHO was normal as was a CT head scan. What is ng/mL. Sherecovers over the next two weeks with no apparent
the most appropriate next step in management? sequelae. Which of the following laboratory test findingsbest
a. anticoagulation explains the underlying etiology for these events?
b. carotidendarterectomy a. ANCA titer of 1:80
c. clopidogrel b. Antistreptolysin O titer of 1:512
d. corticosteroid treatment c. Blood culture positive for Streptococcus, viridans group
e. no action d. Coxsackie B serologic titer of 1:160
e. Total serum cholesterol of 9.6 mmol/l
15. A 21 year old man with Hypertrophic Cardiomyopathy
presents in clinic with dizzy spells but has not had 20. A74yearoldman presented with acute pain, pallor and absent
anysyncopal episodes. Which of the following, if present, pulses in his right leg. Investigations revealed an embolus
would be indicate an increased risk of sudden cardiac death? in his femoral artery. What is the most likely source of this
a. Asymmetric septal hypertrophy with maximum wall embolus?
thickness of 2.1 cm a. marantic endocarditis
b. Blood Pressure drop of 20mmHg during peak exercise b. paradoxical emboli
tolerance testing c. rheumaticendocardialvegetations
c. Left Ventricular Outflow Tract Gradient of 80 mmHg d. right ventricular thrombi
d. Systolic Anterior Movement of the mitral valve on e. thrombi from an atheromatous aorta
echocardiography
e. Worsening exertional angina 21. Whichof the following concerning congenital heart disease is
correct?
16. Whilstattending the cardiology clinic, the staff nurse measures a. ASD is the commonest malformation at birth
the blood pressure of a 61yearold man,and finds that it is
183/100 mmHg sitting and 190/105 standing. He has a heart b. congenital complete heart block is usually associated with
rate of 81/minute, with anirregularly irregular rhythm. On AntiRoantibodies in the mother
auscultation of the heart, there are no murmurs, but he c. Ebstein’s anomaly is associated with maternal exposure to
has bibasilar crackleson chest examination. Which of the lithium carbonate
following pathological findings is most likely to be present? d. Hypoplastic left heart syndrome is characterised by a large,
a. Left ventricular hypertrophy dilated left ventricle
b. Left atrial myxoma e. Osteogenesisimperfecta is associated with aortic stenosis
c. Occlusive coronary atherosclerosis
22. Which of the following regarding the anatomy of the heart is
d. Corpulmonale true?
e. Mitral regurgitation a. The aortic valve is tricuspid.
17. A 24 year old woman develops infective endocarditis involving b. The ascending aorta is entirely outside the pericardial sac.
the aortic valve. She receives a porcine bioprosthesis because c. The left atrial appendage is identified readily by
of her desire to have children and not to take anticoagulant transthoracic echocardiography.
medication. After ten years,she must have this prosthetic valve
1118 d. The pulmonary trunk lies anterior to the ascending aorta. b. Headache is the usual presenting feature.
e. The right atrium is posterior to the left atrium. c. It is defined as systolic blood pressure above the 99th centile
for age.
23. A patient presents with atrial fibrillation and later they revert d. Abnormalities are frequently seen on DMSA scan.
to sinus rhythm. Under which of the following circumstances
is the patient more likely to remain in sinus rhythm? e. Aortic coarctation is the commonest secondary cause.
a. age> 75 years old 28. A 23 year old male presents with a deep vein thrombosis. He
b. been commenced on warfarin has no past medical history but his mother has suffered from
c. left atrium size > 6 cm on ECHO deep vein thromboses. Which of the following is likely to be
found on haematological assessment?
d. short history of AF
a. Factor V Leiden mutation
e. ventricular rate on presentation of 130 bpm
b. Protein S deficiency
24. A 68 year old man has been very ill for months following the c. Protein C deficiency
onset of chronic liver disease with hepatitis C infection. He d. Antithrombin deficiency
experiences a sudden loss of consciousness and then exhibits
e. Lupus anticoagulant
MCQ's
CARDIOLOGY
b. bone marrow aspiration
35. A 30 year old man presents with a history of transient loss of c. intramuscular vitamin B12 alone
consciousness and palpitations. His ECG shows ventricular
d. intramuscular vitamin B12 and oral folic acid together
tachycardia. Which of the following treatments should be
avoided? e. oral folic acid alone
a. adenosine 41. Which of the following compounds has a vasodilating effect?
b. amiodarone a. Antidiuretic hormone
c. DC cardioversion b. Calcitonin
d. flecainide c. Endothelin
e. verapamil d. Renin
36. A 56 year old male with left ventricular systolic dysfunction e. Somatostatin
was dyspnoeic on climbing stairs but not at rest. The patient
was commenced on ramipril and frusemide. Which one of the 42. Which of the following may be responsible for a hypokalaemic
following drugs would improvethe patient’s prognosis? hypertension ?
a. Amiodarone a. Nonclassical congenital adrenal hyperplasia
b. Amlodipine b. Barter’s syndrome
c. Bisoprolol c. Diabetic nephropathy
d. Digoxin d. Liddle’s syndrome
e. Nitrate therapy e. Type IV renal tubular acidosis
37. A 44 year old man has had no major medical problems 43. A52 year old sales representative is admitted with an inferior
throughout his life, except for arthritis pain involving all myocardial infarction. He receives thrombolysis and makes an
extremities for the past couple of years. He has had worsening uneventful recovery. He is discharged on atenolol, aspirin and
orthopnoea and ankle oedema in the past six months. He orvastatin. He enquires how long after his MI must he wait
is afebrile. There is no chest pain. A chest Xray shows before he is able to drive?
cardiomegaly with boththe past six months. He is afebrile. a. One week
There is no chest pain. A chest Xray shows cardiomegaly b. Two weeks
with bothenlarged left and right heart borders, along with
c. Four weeks
pulmonary oedema. Laboratory test findings include
sodium139 mmol/L, potassium 4.3 mmol/L, urea 7 mmol/L d. Three months
creatinine95 μmol/L, and glucose 8.6 mmol/L. Which ofthe e. Six months
following additional laboratory test findings is he most likely
to have? 44. A35 year old woman presented with a history of intermittent
a. Anticentromereantibody titer of 1:320 lightheadedness. Clinical examination and 1 2lead ECG were
normal. Which of the following, if present on a 24 hour Holter
b. Erythrocyte sedimentation rate of 79 mm/Hr
ECG tracing, would be the most clinically important?
c. Haemoglobin of 10.7 g/dL with MCV of 72 fL
a. Atrial premature beats.
d. Serum ferritin of 3400 pmol/L
b. Profound sleepassociatedbradycardia.
e. Spherocytes in his peripheral blood smear
c. Supraventricular tachycardia.
38. Which of the following is a recognised feature of a beta d. Transient Mobitz type 1 atrioventricular block.
lipoproteinaemia? e. Vertricular premature beats.
a. a high serum cholesterol
45. A 57 year old man develops deep venous thrombosis during
b. palmarxanthomas
a hospitalization for prostatectomy. He exhibits decreased
c. advanced atherosclerotic vascular disease mental status with right hemiplegia, and a CT scan of the
d. abnormal red blood cell morphology head suggests an acute cerebralinfarction in the distribution
of the left middle cerebral artery. A chest Xrayreveals cardiac
e. severe mental retardation
enlargement andprominence of the main pulmonary arteries
39. Which of the following infections is least likely to cause that suggests pulmonary hypertension. His serum troponin I
myocarditis? is<0.4 ng/mL. Which of the following lesions is most likely
tobe present on echocardiography?
a. Coxsackie virus
a. Coarctation of the aorta
b. Diphtheria
b. Dextrocardia
1120 c. Pulmonary stenosis 51. An elderly man with a history of asthma, congestive heart
d. Tetralogy of Fallot failure, and peptic ulcer disease is admitted with bronchospasm
and rapid atrial fibrillation. He recieves frequent nebulised
e. Ventricular septal defect salbutamol and IV digoxin loading, his regular medications
are continued. 24 hours after admission his serum potassium
46. A 60 year old man had a myocardial infarction 6 weeks ago.
is noted to be 2.8 mmol/l.Which of his medications is most
He is taking aspirin 75 mg/day and metoprolol 50 mg 2/day.
likely to have caused this abnormality.
During a routine followup Exercise Test he has a 20 beat run
of nonsustained VT. Heachieved stage 4 of the Bruce protocol a. Digoxin
and 92 % of his target heart rate. The nonsustained VT b. ACE inhibitor
occurredhalfway through Stage 2. ST segments were normal c. Salbutamol
during the study. What is the definitive investigation?
d. Ranitidine
a. Coronary angiography.
e. Spironolactone
b. Echocardiogram.
c. Electrophysiological study. 52. In the diagnosis of rheumatic fever, which of the following
d. Thallium exercise scan. may be helpful?
MCQ's
48. Which of the following antiarrhythmic drugs may be used in 54. A78 year old female is referred by her GP with high blood
the treatment of long QT syndrome? pressure. Over the last three months her bloodpressure is
noted to be around 180/80 mmHg. She has a body mass index
a. Amiodarone
of 25.5kg/m2, is a nonsmoker.There are no features to suggest a
b. Atenolol secondary cause for her hypertension. Which of the following
c. Flecainide is the mostappropriate treatment for her blood pressure?
d. Propofanone a. AlphaBlocker
e. Sotalol b. Angiotensin Converting Enzyme (ACE) Inhibitor
c. Angiotensin Blocker
49. A70 year old male was receiving amiodarone 200 mg daily
for intermittent atrial fibrillation. However, he was aware d. Betablocker
of tiredness and lethargy. He appeared clinically euthyroid e. Calcium channel blocker
with no palpable goitre. Investigations revealed: Serum free
T4 23pmol/L (926)Serum total T3 0.8 nmol/L (0.92.8)Serum 55. A 17 year old woman loses consciousness while out jogging
TSH 8.2 mU/L (<5) Which of the following statements would one afternoon, as she has done for many years. She is taken
explain these results? to where a chest Xray,CT brain scan, FBC, and biochemistry
areall normal. Over the next year, she develops mild dyspnea
a. Abnormal thyroxine binding globulin
and fatigue. There are several episodes of presyncope.After
b. Amiodaroneinducedhypothyroidism another syncopal episode, she is referred to a cardiologist
c. ‘sickeuthyroid’ syndrome who orders and ECG that shows changes of left ventricular
d. Spontaneous hypothyroidism hypertrophy and broad Q waves. An echocardiogram reveals
left ventricular and septal hypertrophy, small left ventricle,
e. TSH secreting pituitary adenoma and reduced septal excursion. The septum has a “ground
glass” appearance. She then dies suddenly and unexpectedly.
50. A 65 year old woman, a heavy smoker for many years, has had
The microscopic appearance of the septum with trichrome
worsening dyspnoea for the past 5 years, without a significant
stain reveals myofiber disarray. Which of the following
cough. A chest Xrayshows increased lung size along with
conditions is she most likely to have had?
flattening of the diaphragms, consistent with emphysema.
Over the next several years she develops worsening peripheral a. Diabetes mellitus
oedema. BP 115/70 mmHg. Which of the following cardiac b. Hypertrophic cardiomyopathy
findings is most likely to be present? c. Rheumatic heart disease
a. Constrictive pericarditis d. Systemic lupus erythematosus
b. Left ventricular aneurysm e. Viral myocarditis
c. Mitral valve stenosis
d. Nonbacterial thrombotic endocarditis 56. Whichof the following is true regarding the coronary
circulation?
e. Right ventricular hypertrophy
a. Adenosine is the most important mediator of metabolic
vasodilation.
b. Coronary blood flow is independent of myocardial oxygen e. Somatization disorder 1121
consumption due to autoregulation.
c. Coronary blood flow within a normal range of blood 62. A35yearoldhealthy woman has a faint systolic murmur on
pressure is primarily determined by perfusion pressure. physical examination. An echocardiogram is performed, and
she is found to have a bicuspid aortic valve. In explaining the
d. Increased myocardial O2 demand is met primarily by meaning of this finding to her, the most appropriate statement
increasing O2 extraction. is that?
e. The vasodilatory reserve of the epicardium and endocardium a. An aortic valve prosthesis may eventually need to be placed
is equivalent under normal physiologic conditions.
b. Other family members may have the same condition
57. A54 year old man presents with central crushing chest pain. c. She should be treated with a cholesterolloweringagent
Examination is normal. 12 lead ECG shows STsegment d. The problem resulted from past injection drug usage
elevation in leads II, III, aVF and ST depression in V1, V2 and
e. This is one manifestation of an underlying autoimmune
V3. Which coronary artery is occluded?
disease process
a. Circumflex
CARDIOLOGY
b. Left Anterior Descending 63. Whichof the following findings is the most specific for a
c. Left Main Stem diagnosis of myocardial infarction?
69. Which ONE of the following statements is true about the 75. A 60 year old woman is admitted with sudden onset of chest
diastolic Austin Flint murmur? pain and is diagnosed with an acute myocardial infarction.
a. It is associated with a loud first heart sound. Her acute illness is complicated by low blood pressure and
poor tissue perfusion for several days. Her serum lactate
b. It is an early sign of aortic regurgitation becomes elevated. Her serum urea and creatinine are noted to
c. It can be distinguished from the murmur of mitral stenosis be increasing.
by absence of presystolic accentuation
d. It is due to partial closure of the anterior leaflet of the mitral Day 1 Day 2 Day 3
valve
urea (mmol/L) 8 22 30
MCQ's
70. A 28 year old man who is known to have Hypertrophic Granular and hyaline casts are present on microscopic
Cardiomyopathy has an out of hospital cardiac arrest and is urinalysis. The renal lesion that is most likely to be present in
successfully resuscitated. What is the most appropriate mode thissituation is?
of treatment?
a. Acute tubular necrosis
a. Alcohol Septal Ablation
b. Minimal change disease
b. Amiodarone
c. Nodular glomeruloscerosis
c. Beta Blocker
d. Pyelonephritis
d. Implantable Defibrillator
e. Renal vein thrombosis
e. Myomectomy
76. A 45 year old male type 1 diabetic with a number of complex
71. A 14 year old boy presents with fever. Which of the following diabetic gastrointestinal complications is noted to have a PR
might contribute to a diagnosis of rheumaticfever? interval of 0.18s, a QRS duration of 0.1s and a QT interval of
a. The finding of target lesions on the hands. 0.48s on routine ECG. Whichof the following drugs may be
b. The finding of tender nodules in the fingertips. responsible?
CARDIOLOGY
e. Radiofrequency ablation. a. Constriction of the aorta past the ductusarteriosus
b. Lack of development of the spiral septum and partial
81. A 60 year old man’s echocardiogram shows a dilated left
absence of conus musculature
ventricular cavity with the remainder of the other chamber
sizes normal. The most likely diagnosis is which of the c. Shortening and thickening of chordae tendineae of the
following? mitral valve
a. aortic regurgitation d. Single large atrioventricular valve
b. aortic stenosis e. Supravalvular narrowing in the aortic root
c. hypertensive heart disease 88. Elevation of the jugular venous pressure during inspiration is
d. mitral regurgitation most likely to be found in which of the following situations?
e. mitral stenosis a. a normal physical exam
b. cardiactamponade
82. Left axis deviation is seen on the ECG in which of the
following conditions? c. constrictive pericarditis
a. atrioventricular canal defects. d. dilated cardiomyopathy
b. Ebstein’s anomaly. e. myocarditis
c. large ventricular septal defect. 89. 75 year old man with a history of anterior MI is taking
d. patentductusarteriosus. amiodarone 400 mg/day for history of VT. He has a prolonged
e. tetralogy of Fallot. QT interval on his ECG.What is the most appropriate
management?
83. Whichof the following is true regarding the action of a. Admit to hospital for monitoring.
Clopidogrel?
b. Atenolol.
a. It inhibits cyclooxygenase
c. Change amiodarone to flecainide.
b. It is an ADP receptor antagonist
d. Continue with amiodarone.
c. It is a glycoprotein IIb/IIIa inhibitor
e. Discontinue amiodarone immediately.
d. It is a selective factor Xa inhibitor
e. It is Hydroxymethyl Coenzyme A inhibitor 90. A58 year old man has had an enlarging abdomen for several
months. He has experienced no abdominal orchest pain. On
84. Which ONE of the following is associated with Marfan’s physical examination he has a nontenderabdomen with no
syndrome? masses palpable, but there is a fluid thrill. An abdominal
a. Autosomal recessive inheritance Ultrasound Scan shows a large abdominal fluid collection
with a small cirrhotic liver. A chest Xray shows a globally
b. increased upper : lower body ratio enlarged heart. Which of the following conditions is most
c. Mental retardation likely to be present?
d. Pulmonary stenosis a. Dilated cardiomyopathy
e. Retinal detachment b. Lymphocytic myocarditis
c. Myocardial amyloid deposition
85. A 30 year old intravenous drug abuser develops acute aortic
regurgitation due to infective endocarditis. Which of the d. Nonbacterial thrombotic endocarditis
following is least likely to be found on clinical examination? e. Severe occlusive coronary atherosclerosis
a. decreased cardiac output
91. Anginadue to an imbalance between O2 supply and demand
b. decrescendo diastolic murmur without atherosclerosis would most likely be seen in which of
c. hypotension the following circumstances?
d. mitral valve preclosure a. aortic regurgitation
e. peripheral vasodilatation b. cardiactamponade
c. pulmonary regurgitation
86. A67 year old man presents with sudden onset atrial fibrillation
(ventricular rate of 150/minute). His serumcreatinine d. right heart failure
concentration was 250 umol/L (70110). What is the main factor e. tricuspid regurgitation
that determines the choiceof loading dose of digoxin in this
patient?
a. Absorption
1124 92. An 18 year old man had repeated episodes of breathlessness d. Spironolactone
and palpitations, lasting about 20 minutes and resolving e. Valsartan
gradually. There were no abnormal physical signs. What is the
most likely cause of these features? 98. A40yearoldman received an orthotopic cardiac transplant 7
a. Drug abuse years ago to treat a dilatedcardiomyopathy. Since that time he
b. Panic disorder has been healthy, with no episodes of rejection or infection.
Over the nextyear, however, he develops fatigue with
c. Paroxysmal supraventricular tachycardia exercise. He has worsening pedal edema and orthopnea. On
d. Personality disorder physicalexamination, his vital signs are Temperature 36.3°C,
e. Thyrotoxicosis Pulse 78, Respiratory rate 16, and BP 130/70 mm Hg.There are
no murmurs, rubs, or gallops audible. Bibasilar crackles in the
93. Apreviously well 60 year old lady is admitted with an Acute lungs are audible. Which of thefollowing conditions is most
Anterior Myocardial Infarction. A random blood glucose likely to account for these findings?
concentration was found to be 12.1 mmol/L (<6.7). What is the a. Angiosarcoma
optimal management of her blood sugar? b. Coronary arteriopathy
a. Diet c. Mitral valvular stenosis
MCQ's
b. Gliclazide d. Myocarditis
c. Intravenous insulin plus dextrose e. Pulmonary hypertension
d. Metformin
e. Subcutaneous insulin 99. Whichof the following statements concering the treatment of
acute myocardial infarction is correct?
94. A65yearoldwas advised to start oral digoxin at a dose of 250 a. A pansystolic murmur developing within the first 24 hours
μg daily. His physician explained that the full effect of this does not require further investigation.
treatment would not be apparent for at least a week. Which b. Dipyridamole therapy reduces reinfarction within the first
one of the following pharmacokinetic variables did the year.
physician use to give this explanation?
c. Heparin is beneficial if given with streptokinase.
a. bioavailablity
d. Prophylactic lignocaine given in the first 48 hours is effective
b. halflife in preventing ventricular fibrillation
c. plasma protein binding e. Treatment with a dihydropyridine calcium antagonist is
d. renal clearance associated with increased cardiovascular mortality.
e. volume of distribution
100. A70yearoldman is admitted with an acute Qwaveinferior
95. A75 yearoldlady presents with sudden breathlessness and Myocardial Infarction. On day 5, he suddenly develops
palpitations. On examination, she was observed to have pulmonary oedema and a loud systolic murmur. Which of
an irregular heart beat with rate of 140 bpm, BP 150/84 and the following would be the most useful in establishing a
normal heart sounds. On auscultation of the chest, Fine diagnosis?
basal crepitations are heard. An ECG confirms AF and an old a. chestXray
inferior MI. She is anticoagulated with heparin and given b. coronary arteriography
diuretics. Her heart rate remains rapid. What is the most
appropriate management of the lady’s AF? c. ECG
a. DCCV. d. right heart catheterisation and oximetry
b. IV amiodarone. e. serum cardiac enzymes
c. IV betablocker. 101. A65yearoldman has longstanding stable heart failure treated
d. IV digoxin. with frusemide and enalapril. He complains of swelling
e. Oral quinidine therapy. in his left knee and his GP treats him with Rofecoxib, a
cyclooxygenase2(COX2)inhibitor. Two weeks later the patient
96. A45 year old female presents with a two day history of fever has increasing breathlessness and ankle oedema.Which one of
and joint pains. She has a past history of hypertension for the following effects of rofecoxib is the most likely to explain
which she is receiving antihypertensives.On examination his symptoms?
she has a temperature of 38 Celsius, a facial rash and slight a. decreased absorption of frusemide from the gut
swelling with tenderness of the wrist and ankle joints. Which b. decreased myocardial contractility
of the following Antihypertensivesmay be responsible for her
presentation. c. reduced effective action of enalapril
a. Minoxidil d. the onset of anaemia
b. Phenoxybenzamine e. the onset of fluid retention
c. Hydrallazine 102. Whichof the following statements are true of coronary artery
d. Alphamethyldopa anatomy?
e. Bendrofluazide a. Right bundle branch block in acute anterior myocardial
infarction suggests obstruction prior to the first septal
97. A70yearoldman with dilated cardiomyopathy remains branch ofthe left anterior descending coronaryartery
symptomatic in NYHA class 2 due to chronic heart failure. On b. the posterior descending artery is usually a branch of the
examination his pulse is 90 regular, BP 140/90, heart sounds circumflex artery
normal, chest auscultation did not reveal any abnormalities.
He is currently taking Lisinopril 30 mg OD and Frusemide 80 c. The sinus node is supplied by a branch of the right coronary
mg OD. What is the best treatment option? in over 90% of subjects.
a. Amiodarone d. The AV node is supplied by the left anterior descending
coronary artery.
b. Carvedilol
e. The left main stem is about 4 cm long
c. Digoxin
103. A34 year old male presents with episodes of breathlessness on 109. A14yearoldboy presents with hypertension. Which of the 1125
exertion. Examination reveals a loud P2 and fixed splitting of following statements concerning hypertension in the young is
the second sound. Which of the following may be responsible true?
for these signs? a. Sodium nitroprusside is useful for the longtermtreatment of
a. Maternal chicken pox infection severe cases.
b. Maternal thalidomide therapy b. Headache is the usual presenting feature.
c. 47 XXY karyotype c. It is defined as systolic blood pressure above the 99th centile
d. Homocystinuria for age.
e. Excess maternal alcohol consumption d. Abnormalities are frequently seen on DMSA scan.
e. Aortic coarctation is the commonest secondary cause.
104. A72yearoldman presents with an episode of collapse.
He has had two similar episodes recently, each lasting 110. Anew antihypertensive drug needs to be investigated
about one minute. Four years ago he suffered an anterior to establish its relative potency. Which of the following
myocardial infarction. On examination he was orientated techniques is most appropriate for this purpose?
CARDIOLOGY
and symptomfreewith a regular pulse rate of 80 bpm, BP a. bioassay
140/80 mmHg and the apex beat was displaced to the left.
b. casecontrolstudy
There was an apical systolic murmur. There were no signs of
trauma. ECG showed sinus rhythm, Q waves and ST c. doubleblind,randomized, placebo controlled study
segment elevation anteriorly without reciprocal depression. d. postmarketing surveillance
What is the diagnosis?
e. sequential trial
a. acute anterior myocardial infarction
b. cerebrocasvular accident 111. A53yearoldman presented with hypertension of 150/110 and
is found to have the following results on investigation. Raised
c. epileptic seizure
serum sodium, raised urinary potassium excretion and normal
d. pulmonary embolism serum renin. What is the likely diagnosis?
e. ventricular tachycardia a. Adrenocortical adenoma
b. Coarctation of aorta
105. Deficiencyof which one of the following trace elements is
implicated as a cause of cardiomyopathy? c. Malignant hypertension
a. chromium d. Pheochromocytoma
b. copper e. Renal tumour
c. manganese
112. A29yearoldfemale who is 22 weeks pregnant is noted to have a
d. selenium blood pressure of 150/90 mmHg on 3 separate occasions. Urine
e. zinc protein is negative. Which of the following would be the first
line treatment?
106. A40yearoldman attending a routing screening has a blood a. alpha Methyl Dopa
pressure of 166/100 mmHg. Two weeks later his blood
b. Atenolol
pressure was 150/90 mmHg. He does not smoke. He drinks 35
units alcohol / week. His body mass index (BMI) is 30 kg/m2 c. Magnesium Sulphate
(20 25). What is the best management strategy? d. Nifedipine
a. amlodipine e. Salbutamol
b. atenolol
113. A58yearoldman presents with sudden onset chest pain. He
c. bendrofluazide
has a known history of ischaemic heart disease. ECG shows
d. enalapril ST segment elevation in V1V5without reciprocal depression.
e. lifestyle advice In which territory is the infarction most likely to have take
place?
107. A78 year old female is referred by her GP with high blood a. Anterior
pressure. Over the last three months her blood pressure is
b. Inferior
noted to be around 180/80 mmHg. She has a body mass index
of 25.5kg/m2, is a nonsmoker.There are no features to suggest a c. Lateral
secondary cause for her hypertension. Which of the following d. Inferiolateral
is the mostappropriate treatment for her blood pressure?
e. Posterior
a. AlphaBlocker
b. Angiotensin Converting Enzyme (ACE) Inhibitor 114. An54yearoldmale redevelops chest pain 72 hours after
treatment for an anterior myocardial infarction. Which of
c. Angiotensin Blocker
the following markers will be the most sensitive in detecting
d. Betablocker reinfarction?
e. Calcium channel blocker a. CKMB
b. LDH
108. Whichof the following compounds has a vasodilating effect?
c. Myoglobin
a. Antidiuretic hormone
d. Troponin I
b. Calcitonin
e. Troponin T
c. Endothelin
d. Renin 115. Relationship between arterial blood pressure (BP), cardiac
e. Somatostatin outpur (CO) and peripheral vascular resistance (PVR) can be
described as
a. BP = COxPVR
b. BP = CO/PVR
1126 c. BP = PVR/CO a. Diazepam
d. None of the above b. Fluoxetine
c. Imipramine
116. If a fibrinolytic drug is used for treatment of acute myocardial
infarction, the adverse drug effect that is most likely to occur is d. Lithium
a. Acute renal failure e. Potassium
b. Development of antiplatelet antibodies 124. A hypertensive patient has been using nifedipine for some
c. Encephalitis secondary to liver dysfunction time without untoward effects. If he experiences a rapidly
d. Hemorrhagic stroke developing enhancement of the antihypertensiveeffect of the
drug, it is probably due to
e. Neutropenia
a. Concomitant use of antacids
117. Increased serum levels of which of the following may be b. Fods containing tyramine
associated with a decreased risk of atherosclerosis? c. Grapefruit juice
a. Very lowdensitylipoproteins (VLDL) d. Induction of drug metabolism
b. Lowdensitylipoproteins (LDL)
MCQ's
CARDIOLOGY
d. All of the above chemically as a glycoside?
a. Nifedipine
133. In patients with chronic granulomatous disease which of the
following agents increases the synthesis of tumor necrosis b. Digoxin
factor, leading to activation of phagocytosis? c. Flecainide
a. Aldesleukin d. Cholestyramine
b. Cyclosporine e. Warfarin
c. Filgrastim
141. Inhibition of carbonic anhydrase results in
d. Infliximab
a. Abolition of NaHCO3 reabsorption in proximal tubule
e. Interferon gamma
b. Enhanced of NaHCO3 reabsorption in proximal tubule
134. The mechanism of action of cyclosporine involves c. Enhanced NAHCO3 secretion in distal tubule
a. Activation of calcineurin d. None of the above
b. Binding to cyclophilin to cause inhibition of a cytoplasmic
142. Which of the following cyclotron produced
phosphatase
radiopharmaceuticals is used for assessing regional
c. Blockade of interleukin – 2receptors myocardial perfusion as part of an exercise stress test?
d. Inhibition of phospholipase A2 a. Thallous chloride 201TI USP
e. Suppression of bone marrow progenitors b. Sodium iodide 123I
135. Which one of the following drugs predictably prolongs the PR c. Gallium citrate 67Ga USP
interval and increases cardiac contractility? d. Indium 111In pentetate
a. Digoxin e. Cobalt 57Co cyanocobalamin
b. Lidocaine
143. Mary has a family history of heart disease and wonders if
c. Propranolol garlic would be beneficial to her. Which of the following
d. Quinidine statements is correct about garlic?
e. Verapamil a. Entericcoatedtablets release their contents in the stomach
b. Side effects include heartburn, flatulence, and sweating
136. Which of the following is the drug of choice for management
of cardiac arrhythmias that occur in digitalis toxicity? c. The safety of garlic in pregnancy is unknown
a. Amiodarone d. Garlic does not interact with warfarin
b. Lidocaine 144. Exertion–induced angina, which is relieved by rest,
c. Propranolol nitroglycerin, or both, is referred to as
d. Sotalol a. Prinzmetal’s angina
e. Prazosin b. Unstable angina
c. Classic angina
137. A 54yearoldwoman with severe hypercholesterolemia is to be
treated with a combination of niacin and atorvastatin. With d. Variant angina
this drug combination, it is important that the patient be e. Preinfarction angina
monitored closely for signs of
145. Myocardial oxygen demand is increased by all of the following
a. Agranulocytosis
factors except
b. Gallstones
a. Exercise
c. Lactic acidosis
b. Smoking
d. Myopathy
c. Cold temperatures
e. Thyrotoxicosis
d. Isoproterenol
138. Regarding verapamil, which one of the following statements e. Propranolol
is false?
146. Which of the following agents used in prinzmetal’s angina
a. Angina pectoris is an important indication for the use of
has spasmolytic actions, which increase
verapamil
coronary blood supply?
b. Contraindicated in the asthmatic patient
a. Nitroglycerin
c. Relaxes vascular smooth muscle
b. Nifedipine
1128 c. Timolol a. Lbutilide
d. Isosorbidemononitrate b. Mexiletine
e. Propranolol c. Diltiazem
d. Quinidine
147. The oral absorption of following osmotic diuretic is negligible
e. Propranolol
a. Glycerin
b. Mannitol 156. Which of the following agents has a direct effect on the AV
c. Isosorbide mode, delaying calciumchannel depolarization?
d. All of the above a. Lidocaine
b. Diltiazem
148. Maximal medical therapy for treating angina pectoris is c. Bretylium
represented by which of the following choices?
d. Quinidine
a. Diltiazem, verapamil, nitroglycerin
e. Lbutilide
b. Atenolol, isoproterenol, diltiazem
MCQ's
c. Verapamil, nifedipine, propranolol 157. Which of the following groups of symptoms is most often
d. Isosorbide, atenolol, diltiazem associated with a patient who has rightsided heart failure?
e. Nitroglycerin, isosorbide, atenolol a. Nocturia, rales, paroxysmal nocturnal dyspnea
b. Paroxysmal nocturnal dyspnea, pedal edema, jugular
149. The term ischemic heart disease (IHD) is used to designate all venous distention, hepatojugular reflux
of the following conditions except c. Jugular venous distention, hepatojugular reflux, pedal
a. Angina pectoris edema, shortness of breath
b. Sudden cardiac death d. Hepatojugular reflux, jugular venous distension, pedal
c. Congestive heart failur (CHF) edema, abdominal distention
d. Arrhythmias e. Paroxysmal nocturnal dyspnea, jugular venous distention,
abdominal distention, shortness of breath
150. Which of the following thrombolytic agents would be
appropriate at this time? 158. Which of the following combinations of drugs, when used
together, reduce both preload and afterload?
a. Anisoylated plasminogen streptokinase activator complex
(APSAC) a. Nitroglycerin and isosorbidedinitrate
b. Streptokinase (SK) b. Hydralazine and isosorbidedinitrate
c. Recombinant tissuetypeplasminogen activator (tPA) c. Captopril and methyldopa
d. Prazosin and angiotension II
151. Strong anticholinergic effects limit the antiarrhythmic use of
e. Hydralazine and methyldopa
a. Quinidine
b. Procainamide 159. When digoxin is used in a patient with congestive heart failure
(CHF), it works by exerting a positive effect on
c. Tocainide
a. Stroke volume
d. Flecainide
b. Total peripheral resistance
e. Disopyramide
c. Heart rate
152. Following loop diuretic is a phynoxy acetic acid derivative d. Blood pressure
a. Furosemide e. Venous return
b. Bumetanide
160. Because of proven beneficial effects on “cardiac remodeling”,
c. Ethacrynic acid
these agents are now indicated as first line therapy in CHF
d. All of the above patients. Which of the following isrepresentative of this group
of drugs?
153. Following potassium sparing diuretic is a mineralocorticoid
receptor antagonist a. Hydrochlorothiazide
a. Amiloride b. Enalapril
b. Triamterene c. Furosemide
c. Spironolactone d. Carvedilol
d. All of the above e. Bumetanide
154. A patient receiving a class I antiarrhythmic agent on a chronic 161. For treating the patient with congestive heart failure (CHF),
basis complains of fatigue, lowgrade fever,and joint pain which of the following dosages of dopamine is selected for its
suggestive of systemic lupus erythematosus (SLE). The patient positive inotropic effects?
is most likely receiving a. 2.0 mg/kg/min
a. Lidocaine b. 5–10 mg/kg/min
b. Procainamide c. 10–20 mg/kg/min
c. Quinidine d. 40 mg/kg/min
d. Flecainide e. 40 mg/kg/min
e. Propranolol
162. Milrinone is an example of
155. Which of the following drugs is a class IV antiarrhythmic a. Phosphodiesterase I inhibitor
that is primarily indicated for the treatment of b. Phosphodiesterase II inhibitor
supraventriculartachyarrhythmias?
c. Phosphodiesterase III inhibitor b. Tolerance 1129
d. Phosphodiesterase IV inhibitor c. Addiction
d. Physical dependence
163. Situations that predispose a digitalistreated patient to toxicity
include e. Cummulative poisoning
a. Hypercalcemia 172. The action of digitalis is enhanced by
b. Hyperkalemia a. Sodium
c. Hypermagnesemia b. Calcium
164. Unfractionated heparin binds to antithrombin III and c. Magnesium
inactivates clotting factor(s) d. Potassium
a. Xa e. Chloride
b. Ixa
173. In case of acute pain of angina pectoris the most effective
c. Iia
CARDIOLOGY
treatment would be to administer
d. All of the above a. Mannitolhexanitrate
e. None of the above b. Erythrityltetranitrate
165. A patient to be commenced on oral anticoagulant therapy for c. Sodium nitrate
DVT would be treated with d. Pentaerythritoltetranitrate
a. Oral anticoagulant therapy with warfarin for a goal e. Nitroglycerin
intenational normalized ration (INR) of 2–3
174. Which of the following is used to lower blood lipid levels?
b. Oral anticoagulant therapy with warfarin for a goal INR of
2.5–3.5 a. Trimethadione
c. Oral anticoagulant therapy with aspirin for a goal INR of b. Clofibrate
2–3 c. Flucytosine
d. Coumarin
166. A patient on oral anticoagulant therapy is commenced on
sulfamethoxazoletrimethoprim, doublestrength twice daily. e. Propranolol
One may expect to see the international normalized ratio
175. The chief use of levoarterenol is to treat
a. Increase
a. Shock
b. Decrease
b. Diabetes
c. Remain unchanged
c. Hypertension
167. When compared to unfractionated heparin, low molecular d. Cardiac arrhythmias
weight heparins have
e. Iron deficiencies
a. Preferential binding affinity to factor Xa relative to Iia
(thrombin) 176. Tolerance to nitroglycerin may be overcome by
b. Shorter halflives a. Initially using the largest safe dose of the drug
c. Dose – dependent renal clearance b. Using other nitrites
c. Temporarily discontinuing the drug for one or two weeks
168. Acute renal failure (ARF) may be caused by all of the following
except d. Use of higher doses
a. Acute tubular necrosis (ATN) due to drug therapy (e.g., e. None of the above
aminoglycosides, contrast media)
177. Quinidine can cause paradoxical tachycardia in a patient of
b. Severe hypotension or circulatolry collapse
a. Stick sinus syndrome
c. Decreased cardiac output, as from congestive heart failure
b. Auricular extrasystoles
d. Hemolysis, myoglobinuria
c. Auricular fibrillation
e. Hyperkalemia
d. Ventricular extrasystoles
169. During Phase 2 of action potential in cardiac cell, depolarizing
current through calcium channels is 178. Quinidine is now used primarily for
balanced by a. Conversion of auricular fibrillation to sinus rhythm
a. Delayed rectifier potassium current b. Control of ventricular rate in atrial flutter
b. outward chloride channel c. Termination of ventricular tachycardia
c. Both (a) and (b) d. Prevention of recurrences of atrial and ventricular
extrasystoles/tachycardias
d. None of the above
179. Procainamide differs from quinidine in the following respect
170. The action of quinidine differs from that of digitalis in
a. It does not cause paradoxical tachycardia
a. Decreasing irritability of cardiac muscle
b. It has no alfa adrenergic blocking activity
b. Preventing passage of impulses to the ventricle
c. It has little antivagal action
c. Increasing irritability of heart muscle
d. Both (b) and (c)
d. Reducing conductivity
e. None of the above 180. In heart, potassium channels determine
a. Pacemaker function
171. Overuse of digitalis may result in
b. Resting potential
a. Habituation
1130 c. Action potential duration 189. The primary mechanism of beneficial effect of glyceryltrinitrate
d. All of the above in classical angina pectoris is
a. Increase in total coronary blood flow
181. Lidocaine is the preferred antiarrhythmic for emergency b. Redistribution of coronary blood flow
control of cardiac arrhythmias following acute myocardial
infarction because c. Reduction of cardiac preload
a. It has a rapidly developing and titratable antiarrhythmic d. Reduction of cardiac after load
action
190. Enhanced automaticity in cardiac cells may occur because of
b. It casues little myocardial depression and hypotension
a. ßadrenergic stimulation
c. It has broad spectrum antiarrhythmic efficacy in atrial as
b. Hypokalemia
well as ventricular arrhythmias
c. Mechanical stretch of cardiac muscles
d. Both (a) and (b)
d. All of the above
182. Hypothyroidism is a possible consequence of prolonged
therapy with 191. Glyceryltrinitrate is administratered by all of the following
routes except
MCQ's
a. Procainamide
a. Oral
b. Mexiletine
b. Sublingual
c. Sotalol
c. Intramuscular
d. Amioiv
d. Intravenous
183. Which of the following drugs is preferred for termination of
paroxysmal supraventricular tachycardia 192. A patient of acute myocardial infarction being treated
in intensive care unit developed left ventricular failure
a. Digoxin
with raised central venous pressure. It was decided to use
b. Quinidine glyceryltrinitrate. Which route/method of administration
c. Propranolol would be most suitable.
d. Verapamil a. Sublingual
b. Oral
184. The following drug is used to reduce the frequency of angina
pectoris as well as to terminate an acute attack c. Intravenous bolus injection
a. Digoxin d. Slow
b. Furosemide 193. A patient suffers from spisodic pain diffusely localized over
c. Enalapril the chest and upper abdomen, which is relieved by sublingual
d. Amrinone glyceryltrinitrate. He could be suffering from
a. Angina pectoris
185. Antianginal drugs afford the following benefit/benefits b. Biliary colic
a. Terminate anginal attacks c. Esophageal spasm
b. Decrease the frequency of anginal attacks d. All of the above
c. Retard the progression of coronary artery disease
d. Both (a) and (b) 194. The dihydropyridines block the following type of calcium
channels.
186. Choose the correct statement about the action of nitrates on a. Ltypevoltage sensitive channels
coronary vessels b. Ttypevoltage sensitive channels
a. They mitigate angina pectoris by increasing total coronary c. Ntypevoltage sensitive channels
flow
d. Receptor operated calcium channels
b. They preferentially dilate conducting arteries without
affecting resistance arterioles 195. Which of the following drugs is most likely to accentuate
c. They preferentially dilate autoregulatory arterioles without varient (Prinzmetal) angina ?
affecting the larger arteries a. Digoxin
d. They increase subepicardial blood flow without affecting b. Furosemide
subendocardial blood flow
c. Enalapril
187. Organic nitrates relax vascular smooth muscle by d. Amrinone
a. Increasing intracellular cyclic AMP
196. In cardiac cells, adenosine
b. Increasing intracellular cyclic GMP
a. Causes shortening of action potential duration
c. Decreasing intracellular cyclic AMP
b. Depolarization
d. Both (b) and (c)
c. Increase in normal automaticity
188. Select the organic nitrate which undergoes minimal d. All of the above
firstpassmetabolism in the liver
197. Which of the following antianginal drugs is most likely to
a. Glyceryltrinitrate
produce tachycardia as a side effect ?
b. Isosorbidedinitrate
a. Amlodipine
c. Isosorbidemononitrate
b. Nifedipine
d. Erythrityltetranitrate
c. Diltiazem
d. Verapamil
198. Which of the following is not an attribute of amlodipine ? 207. Digoxin given for cardiac failure is extremely valuable in 1131
a. High and consistent oral bioavailability patients of
b. Large volume of distribution a. Thyrotoxicosis
c. Generation of an active metabolite b. Beriberi
d. Long elimination halflife c. Corpulmonale
d. Atrial fibrillation
199. Propranolol should not be prescribed for a patient of angina
pectoris who is already receiving 208. The agent given sublingually in an acute attack of angina
a. Nifedipine pectoris is
b. Felodipine a. Glyceryltrinitrate
c. Verapamil b. Amyl nitrite
d. Isosorbidemononitrate c. Erythritaltetranitrate
d. Pentaerythritoltetranitrate
CARDIOLOGY
200. Which of the following drugs is a potassium channel opener ?
a. Pinacidil 209. Following statement is true about lidocaine
b. Hydralazine a. Reduce the slope of Phase 4
c. Glibenclamide b. Threshold excitability is not altered
d. Amiloride c. Action potential is not affected
d. All of the above
201. Though nitrates and calcium channel blockers are both
vasodilators, they are used concurrently in angina pectoris 210. Aspirin in small doses (50 to 150 mg per day)
because a. Is of benefit in patients of unstable angina
a. They antagonize each other’s side effects b. Has thrombolytic action
b. Nitrates primarily reduce preload while calcium channel c. PREFerentially inhibits prostacyclin synthetase enzyme
blockers primarily reduce after load
d. MAY Alleviate need for Verapamil in variant angina
c. Nitrates increase coronary flow while calcium
d. Both (b) and (c) 211. Major beneficial effect of nitrates in classical angina is due to
a. Dilation of veins more than arteries
202. Coronary steal phenomenon’ has been noted most frequently
b. Increase in total coronary blood flow
with
c. An increase in the end diastolic size of the heart
a. Glyceryltrinitrate
d. An increase in the heart rate
b. Dipyridamole
c. Propranolol 212. Major lipid class present in chylomicrons is
d. Diltiazem a. Endogenous triglycerides
b. Cholesterol esters
203. Which of the following drugs is believed to improve
microcirculation in peripheral vascular diseases by promoting c. Dietary triglycerides
RBC flexibility ? d. All of the above
a. Cyclandelate
213. A 65-year-old man presents with central crushing chest pain
b. Theophyline
for the first time. Heis transferred immediately to the closest
c. Pentoxiphyline cardiac unit to undergo a primarypercutaneous coronary
d. Nicotinic acid intervention. There is thrombosis of the left circumflexartery
only. Angioplasty is carried out and a drug-eluding stent is
204. Higher incidence of myocardial infarction and increased inserted. Whatare the most likely changes to have
mortality has been noted with the use of the following occurred on ECG during admission?
antihypertensive drug a. ST depression in leads V1–4
a. Nifedipine b. ST elevation in leads V1–6
b. Verapamil c. ST depression in leads II, III and AVF
c. Diltiazem d. ST elevation in leads V5–6
d. Lisinopril e. ST elevation in leads II, III and AVF
205. Cardiac glycosides are obtained from following plant source. 214. A 78-year-old woman is admitted with heart failure. The
a. Rauwolfiaserpentina underlying cause isdetermined to be aortic stenosis. Which
b. Strophanthusgratus sign is most likely to be present?
c. Ricinuscommounts. a. Pleural effusion on chest x-ray
d. Atropa belladonna b. Raised jugular venous pressure (JVP)
c. Bilateral pedal oedema
206. Therapeutic dose of digoxin in a normal individual has the d. Bibasalcrepitations
following effects, except
e. Atrial fibrillation
a. Increase in the speed of myocardial contractility
b. No significant change in cardiac output 215. A patient is admitted with pneumonia. A murmur is heard on
c. Relaxation of peripheral vascular bed examination. Whatfinding points to mitral regurgitation?
d. Increase in the force of myocardial contractility a. Murmur louder on inspiration
b. Murmur louder with patient in left lateral position
1132 c. Murmur louder over the right 2nd intercostal space 221. A 49-year-old man is rushed to complaining of a 20-minute
midclavicular line history of severe, crushing chest pain. After giving the patient
d. Corrigan’s sign glyceryltrinitrate (GTN) spray, he is able to tell you he suffers
from hypertension and type 2 diabetes and is allergic to
e. Narrow pulse pressure aspirin. The most appropriate management is:
216. A 79-year-old woman is admitted to the coronary care unit a. Aspirin
(CCU) with unstableangina. She is started on appropriate b. Morphine
medication to reduce her cardiac risk. She ishypertensive, c. Heparin
fasting glucose is normal and cholesterol is 5.2. She is found
to be inatrial fibrillation. What is the most appropriate d. Clopidogrel
treatment? e. Warfarin
a. Aspirin and clopidogrel
222. While on call you are called by a nurse to a patient on the ward
b. Digoxin complaining of light headedness and palpitations. When you
c. Cardioversion arrive the patient is not conscious but has a patent airway and
d. Aspirin alone is breathing with oxygen saturation at 97 per cent. You try to
MCQ's
palpate a pulse but are unable to find the radial or carotid. The
e. Warfarin registrar arrives and after hearing your report of the patient
decides to shock the patient who recovers. What is the patient
217. A 55-year-old man has just arrived in complaining of
most likely to have been suffering?
20minutes of central crushing chest pain. Which feature is
most indicative ofmyocardial infarction at this moment in a. Torsades de Pointes
time? b. Ventricular fibrillation
a. Inverted T waves c. Sustained ventricular tachycardia
b. ST depression d. Non-sustained ventricular tachycardia
c. ST elevation e. Normal heart ventricular tachycardia
d. Q waves
223. A 67-year-old man presents to with a 3-day history of shortness
e. Raised troponin of breath. On examination you palpate the radial pulse and
notice that the patient has an irregular heart beat with an
218. A 66-year-old woman presents to with a 2-day history of
overall rate of 140 bpm. You request an electrocardiogram
shortness of breath. The patient notes becoming progressively
(ECG) which reveals that the patient is in atrial fibrillation.
short of breath as well as a sharp pain in the right side of
Which of the following would you expect to see when
the chest which is most painful when taking a deep breath.
assessing the JVP?
The patient also complains of mild pain in the right leg,
though there is nothing significant on full cardiovascular and a. Raised JVP with normal waveform
respiratory examination. Heart rate is 96 and respiratory rate is b. Large ‘v waves’
12. The patient denies any weight loss or long haul flights but c. Cannon ‘a waves’
mentions undergoing a nasal polypectomy 3 weeks ago. The
most likely diagnosis is: d. Absent ‘a waves’
a. Muscular strain e. Large ‘a waves’
b. Heart failure 224. A 78-year-old woman is admitted to your ward following a
c. Pneumothorax 3-day history of shortness of breath and a productive cough of
d. Angina white frothy sputum. On auscultation of the lungs, you hear
bilateral basal coarse inspiratory crackles. You suspect that
e. Pulmonary embolism the patient is in congestive cardiac failure. You request a chest
x-ray. Which of the following signs is not typically seen on
219. A 59-year-old man presents for a well person check. A
chest x-ray in patients with congestive cardiac failure?
cardiovascular, respiratory, gastrointestinal and neurological
examination is performed. No significant findings are found, a. Lower lobe diversion
except during auscultation a mid systolic click followed by a b. Cardiomegaly
late systolic murmur is heard at the apex. The patient denies c. Pleural effusions
any symptoms. The most likely diagnosis is:
d. Alveolar oedema
a. Barlow syndrome
e. Kerley B lines
b. Austin Flint murmur
c. Patent ductusarteriosus 225. A 56-year-old man presents to your clinic with symptoms
d. Graham Steell murmur of exertional chest tightness which is relieved by rest. You
request an ECG which reveals that the patient has first degree
e. Carey Coombs murmur heart block. Which of the following ECG abnormalities is
typically seen in first degree heart block?
220. A 60-year-old man presents to with a 3-day history of
increasingly severe chest pain. The patient describes the pain a. PR interval >120 ms
as a sharp, tearing pain starting in the centre of his chest and b. PR interval <300 ms
radiating straight through to his back between his shoulder c. PR interval <200 ms
blades. The patient looks in pain but there is no pallor, heart
rate is 95, respiratory rate is 20, temperature 37°C and blood d. PR interval >200 ms
pressure is 155/95 mmHg. The most likely diagnosis is: e. PR interval <120 ms
a. Myocardial infarction
226. You see a 57-year-old woman who presents with worsening
b. Myocardial ischaemia shortness of breath coupled with decreased exercise tolerance.
c. Aortic dissection She had rheumatic fever in her adolescence and suffers from
d. Pulmonary embolism essential hypertension. On examination she has signs which
point to a diagnosis of mitral stenosis. Which of the following
e. Pneumonia is not a clinical sign associated with mitral stenosis?
a. Malar flush hypertensive for about five years and his blood pressure 1133
b. Atrial fibrillation control had been good on three drugs. However, he had
decided to stop all medication two months before this event.
c. Pan-systolic murmur which radiates to axilla Which of the following would be your preferred parenteral
d. Tapping, undisplaced apex beat medication at this point?
e. Right ventricular heave a. Glyceryltrinitrate
b. Hydralazine
227. A 48-year-old woman has been diagnosed with essential
hypertension and was commenced on treatment three months c. Labetalol
ago. She presents to you with a dry cough which has not been d. Sodium nitroprusside
getting better despite taking cough linctus and antibiotics. e. Phentolamine
You assess the patient’s medication history. Which of the
following antihypertensive medications is responsible for the 232. A 16-year-old male is referred for assessment of hypertension.
patient’s symptoms? On average, his blood pressure is 165/85 mmHg, with
a. Amlodipine radiofemoral delay. There is a mid-systolic murmur maximal
CARDIOLOGY
b. Lisinopril at the aortic area, and radiating to the back. Clinical findings
and the ECG are compatible with left ventricular hypertrophy.
c. Bendroflumethiazide What is the most likely underlying pathology?
d. Frusemide a. Hypertrophic obstructive cardiomyopathy
e. Atenolol b. Congenital aortic stenosis
228. A 62-year-old male presents with palpitations, which are c. Coarctation of the aorta
shown on ECG to be atrial fibrillation with a ventricular d. Patent ductusateriosus
rate of approximately 130/minute. He has mild central chest e. Atrial septal defect
discomfort but is not acutely distressed. He first noticed these
about 3 hours before coming to hospital. As far as is known 233. A 16-year-old boy is diagnosed with a small ventricular septal
this is his first episode of this kind. Which of the following defect, having been screened by echocardiography because of
would you prefer as first-line therapy? a family history of hypertrophic obstructive cardiomyopathy.
a. Anticoagulate with heparin and start digoxin at standard He is entirely asymptomatic, plays several sports regularly and
daily dose has no growth retardation. The echocardiogram also confirms
b. Attempt DC cardioversion a small left to right shunt, with pulmonary to systemic flow
ratio only just above one. Which of the following is the most
c. Administer bisoprolol and verapamil, and give warfarin likely to be a significant complication of his condition?
d. Attempt cardioversion with IV flecainide a. Pulmonary hypertension
e. Wait to see if there is spontaneous reversion to sinus rhythm b. Heart failure
229. A 76-year-old male is brought to after collapsing at home. c. Dysrhythmias
He has recovered within minutes and is fully alert and d. Endocarditis
orientated. He says this is the first such episode that he e. Shunt reversal (right to left flow)
has experienced, but describes some increasing shortness of
breath in the previous six months and brief periods of central 234. A 52 year-old woman has been treated for several years
chest pain, often at the same time. On examination, blood with amlodipine and lisinopril for what has been presumed
pressure is 115/88 mmHg and there are a few rales at both to be primary hypertension. She is seen by her GP having
bases. On ECG there are borderline criteria for left ventricular complained of persistent left loin pain. Her BP is 150/95
hypertrophy. Which of the following might you expect to find mmHg. She is tender in the left loin and both kidneys appear
on auscultation? to be enlarged. On urine dipstick testing, there is microscopic
a. Mid-diastolic murmur best heard at the apex haematuria. Which of the following is likely to be the most
b. Crescendo systolic murmur best heard at the right sternal appropriate investigation at this point?
edge a. Urinary tract ultrasound
c. Diastolic murmur best heard at the left sternal edge b. Abdominal and pelvic computed tomography (CT) scan
d. Pan-systolic murmur best heard at the apex c. Microscopy of the urine (microbial and cytological)
e. Pan-systolic murmur best heard at the left sternal edge d. Renal biopsy
e. Intravenous urogram
230. A 63-year-old male was admitted to 2 days after discharge
following an apparently uncomplicated MI. He complained 235. A 61-year-old man presents with a 2-hour history of moderately
of rapidly worsening shortness of breath over the previous severe retrosternal chest pain, which does not radiate and is
48 hours but no further chest pain. He was tachypnoeic and not affected by respiration or posture. He complains of general
had a regular pulse of 110/minute, which proved to be sinus malaise and nausea, but has not vomited. His ECG shows ST
tachycardia. The jugular venous pressure was raised and a segment depression and T wave inversion in the inferior leads.
pan-systolic murmur was noted, maximal at the left sternal Troponin levels are not elevated. He has already been given
edge. Which of the following is the most likely diagnosis? oxygen, aspirin and intravenous GTN; he is an occasional user
a. Mitral incompetence of sublingual GTN and takes regular bisoprolol for stable
b. Ventricular septal defect angina. What would be the most appropriate next step in his
management?
c. Aortic stenosis
a. IV low-molecular weight heparin
d. Dressler’s syndrome
b. Thrombolysis with alteplase
e. Further myocardial infarction
c. IV nicardapine
231. A 57-year-old male is admitted complaining of headaches d. Angiography with stenting
and blurring of vision. His blood pressure is found to be e. Oral clopidogrel
240/150 mmHg and he has bilateral papilloedema, but is
fully orientated and coherent. He had been known to be
1134 236. A 41-year-old woman is referred for assessment after suffering lasting over 2 hours. Her pulse rate is 84/minute in sinus
a second pulmonary embolus within a year. She has not rhythm, and blood pressure is 134/86 mmHg. The ECG shows
been travelling recently, has not had any surgery, does not anterior ST segment elevation, but troponin levels do not rise.
smoke and does not take the oral contraceptive pill. She is not Subsequent coronary angiography is normal. What is the most
currently on any medication as the diagnosis is retrospective likely diagnosis?
and she is now asymptomatic. What should be the next step in a. MI
her management?
b. Stable angina
a. Initiation of warfarin therapy
c. Unstable angina
b. ECG
d. Anxiety
c. Thrombophilia screen
e. Variant angina
d. Insertion of inferior vena cava filter
e. Duplex scan of lower limb veins and pelvic utrasound 242. A previously fit 19-year-old man presents with unusual
shortness of breath on exertion. At times, this is also associated
237. A 32-year-old woman attends her GP for a routine medical with central chest pain. On examination there is a loud mid-
examination and is noted to have a mid-diastolic murmur systolic murmur at the left sternal edge. Heart rate and blood
MCQ's
with an opening snap. Her blood pressure is 118/71 mmHg pressure are normal and there is no oedema. The ECG shows
and the pulse is regular at 66 beats per minute. She is entirely left axis deviation and the voltage criteria for left ventricular
asymptomatic and chest x-ray and ECG are normal. What hypertrophy and the echocardiogram reveals a significant
would be the most appropriate investigation at this point? thickened interventricular septum, with delayed ventricular
a. ECG filling during diastole. There is a family history of sudden
death below the age of 50. Which of the following would be
b. Anti-streptolysin O titre your initial therapy?
c. Cardiac catheterization a. Digoxin
d. Thallium radionuclide scanning b. Long-acting nitrates
e. Colour Doppler scanning c. Beta-blockers
238. A 46-year-old man develops sudden severe central chest pain d. Rate-limiting calcium channel blockers
after lifting heavy cases while moving house. The pain radiates e. Partial excision of the septum
to the back and both shoulders but not to either arm. His BP is
155/90 mmHg, pulse rate is 92 beats per minute and the ECG is 243. A 44-year-old woman presents with episodes of headaches,
normal. He is distressed and sweaty, but not nauseated. associated with anxiety, sweating and a slow pulse rate. At
What would you consider the most likely diagnosis? the time of her initial consultation, her blood pressure was
a. Pneumothorax 150/95 mmHg seated, but 24 hour ambulatory monitoring
shows a peak of 215/130 mmHg, associated with the symptoms
b. MI described above. Which of the following would be your initial
c. Pulmonary embolism diagnostic procedure?
d. Aortic dissection a. Magnetic resonance imaging (MRI) scans of the abdomen
e. Musculoskeletal pain and pelvis
b. Measurement of random plasma catecholamines
239. A 49-year-old woman presents with increasing shortness of
c. Measurement of urinary metanephrines over several 24
breath on exertion developing over the past three months. She
hour periods
has no chest pain or cough, and has noticed no ankle swelling.
On examination, blood pressure is 158/61 mmHg, pulse is d. Glucose tolerance test
regular at 88 beats per minute and there are crackles at both e. Pharmacological provocation using clonidine
lung bases. There is a decrescendo diastolic murmur at the left
sternal edge. What is the most likely diagnosis? 244. A 56-year-old man presents to the department with a 2-hour
a. Aortic regurgitation history of central chest pain radiating to the left arm. He is
anxious, nauseated and sweaty. His pulse rate is 120/minute
b. Aortic stenosis in sinus rhythm and the ECG reveals ST elevation in leads II,
c. Mitral regurgitation III and aVF. The troponin level is significantly raised. This is
d. Mitral stenosis certainly acute MI. Which is the most likely coronary vessel to
be occluded?
e. Tricuspid regurgitation
a. Circumflex artery
240. A 21-year-old man is on his way home from a party when he b. Left anterior descending artery
experiences the sudden onset of rapid palpitations. He feels
c. Right coronary artery
uncomfortable but not short of breath and has no chest pain.
He goes to the nearest department, where he is found to have d. Left main coronary artery
a supraventricular tachycardia (SVT) at a rate of 170/minute. e. Posterior descending artery
Carotid sinus massage produced transient reversion to sinus
rhythm, after which the tachycardia resumed. What would be 245. A 45-year-old woman complains of increasing shortness of
the next step in your management? breath on exertion, as well as orthopnoea, for the previous 3–4
a. Repeat carotid sinus massage months. She had apparently recovered from pericarditis about
a year earlier. On ECG there is low voltage, especially in the
b. IV verapamil limb leads, and the chest x-ray shows pericardial calcification.
c. IV propranolol The presumptive diagnosis is constrictive pericarditis. Which
d. IV adenosine of the following physical signs would be consistent with this?
e. Synchronized DC cardioversion a. Increased jugular distention on inspiration
b. Third heart sound
241. A 44-year-old woman attends her local department with a
c. Fourth heart sound
history of at least six months of frequent central chest pain
in the early morning or during the night. She had no chest d. Rales at both lung bases
pain on exertion. This had been a particularly severe attack, e. Loud first and second heart sounds
246. A 71-year-old man is being treated for congestive heart failure atrial fibrillation. Which of the following statements is correct 1135
with a combination of drugs. He complains of nausea and regarding the management of this patient?
anorexia, and has been puzzled by observing yellow rings a. Digoxin effectively prevents recurrence of the arrhythmia
around lights. His pulse rate is 53/minute and irregular and
blood pressure is 128/61 mmHg. Which of the following b. Anticoagulation is not necessary
medications is likely to be responsible for these symptoms? c. Sotalol may be effective
a. Lisinopril d. Amiodarone should be avoided
b. Spironolactone e. Flecainide orally may be an effective as-needed treatment to
c. Digoxin abort anAttack
CARDIOLOGY
though without dieting. On examination, her pulse rate is be your next stage in his management?
approximately 120/min and irregularly irregular. Her blood a. Arrange for his medication to be given under direct
pressure is 142/89 mmHg and her body mass index is 19. There observation
are no added cardiac sounds. The ECG confirms the diagnosis
of atrial fibrillation. What would you suggest as the most b. Add spironolactone to his medication
useful next investigation. c. Arrange urinary catecholamine assays
a. Thyroid function tests (TSH, free T4) d. Request an adrenal CT scan
b. ECG e. Add verapamil to his medication
c. Chest x-ray
253. Cardiac toxicity is seen most in
d. Full blood count
a. Bleomycin
e. Fasting blood sugar
b. Adriamycin
248. A 58-year-old man has made an excellent functional recovery c. Methotrexate
after an anterior MI. He is entirely asymptomatic and there d. Busulphan
is no abnormality on physical examination. His blood
pressure is 134/78 mmHg and he is undertaking a cardiac 254. Cardiospasm is
rehabilitation programme. Which of the following would you a. Coronary artery stenosis
not recommend as part of his secondary prevention planning?
b. Presbyoesophagus
a. Aspirin
c. Esophageal webs
b. Lisinopril
d. Cricoid narrowing
c. Simvastatin
d. Bisoprolol 255. Carotid sinus massage produces
e. Omega-3 fatty acids a. Reflex bradypnea
b. Reflex bradycardia
249. A 25-year-old woman with known mitral valve prolapse
develops a low grade fever, malaise and night sweats within c. Reflex tachycardia
a couple of weeks of a major dental procedure. Examination d. Hyperpnea
reveals a pulse rate of 10/minute, which is regular, tender
vasculitic lesions on the finger pulps and microscopic 256. To differentiate between supraventricular tachycardia
haematuria. Which investigation is most likely to provide a and ventricular tachycardia of aberrant conduction all the
definitive diagnosis? following points support the latter except
a. Full blood count a. Wide QRS >0.16 secs
b. ECG b. Carotid massage causes a 2:1 block
c. Autoantibody screen c. Oesophageal leads show A-V dissociation
d. Blood culture
257. Linked angina means..
e. Coronary angiography
a. Tietz syndrome
250. An asymptomatic 31-year-old woman has been referred for b. Viral pericarditis
cardiological assessment. After her ECG she was told that she c. Variant angina
had mitral valve prolapse and would like further information
d. Angina caused by myocardicalischaemia due to
on this condition. Which of the following statements is
Gastrointestinal disturbances.
correct?
a. Beta-blocker therapy is indicated 258. Any visible edema of both lower extremetied each preceded
b. Angiotensin-converting enzyme (ACE) inhibitor therapy is by weight gain of
indicated a. 0.5-1kg
c. One or both leaflets of the mitral valve are pushed back into b. 1-2kg
the leftatrium during systole c. 3-5kg
d. Significant mitral regurgitation will eventually develop d. 5-7kg
e. Exercise should be restricted
259. In pheochromocytoma there is increased level of
251. A 69-year-old woman complains of intermittent palpitations, a. Serum HMA
lasting several hours, which then stop spontaneously. She also
suffers from asthma. Holter monitoring confirms paroxysmal b. Serum bradykinin
1136 c. Urinary VMA 270. Diastasis phase accounts for ------% of ventricular filling
d. All of the above a. 80%
b. 70%
260. Pulsatile exophthalmos is seen in
c. 60%
a. Severe AR
d. <5%
b. SEVERE TR
c. TR 271. Which are the values of Troponin T that are prognostically
d. MR significant?
a. >/= to 0.05 ng/ml
261. Osteogenesisimperfecta may be associated with all except: b. >/= to 0.1 ng/ml
a. Aortic dialation c. >/= to 0.001 ng/ml
b. AR d. >/= to 0.01 ng/ml
c. MVP
d. PR 272. Most consistant phase of pericardial rub is
MCQ's
a. Systolic phase
262. Kussamaul’s sign is b. Presystolic
a. Paradoxical rise in height of JVP c. Mid diastolic
b. Paradoxical decrease in height of the JVP d. All of the above
c. Seen in AS
d. Typically occurs in cardiac tamponade 273. The most common cause of secondary hypertension in
children is
263. W shaped JVP is seen in a. Renal artery stenosis
a. Constrictive pericarditis b. Adrenal tumors
b. RCM c. Renal disease
c. DCM d. Coarctation of aorta
d. HOCM
274. The duration of the depolarisation of the heart from the SA
264. Dicrotic notch in normal pulse indicated node to the AV node is reflected on the ECG by
a. Opening of mitral valve a. QT interval
b. Opening of aortic valve b. PR interval
c. Closure of MV c. RS interval
d. Closire of AV d. RR interval
265. Pulsustardus means 275. Digoxin toxicity can be recognized when there is
a. Late peaking a. QT interval is shortened
b. Slow rising b. Ventricular bigeminy
c. Reduced peak c. Atrial flutter
d. Small amplitude d. Mobitz II degree heart block
266. Visible pulsation of the retinal artery is known as 276. Classical S1, Q3, T3 PATTERN occurs in------ % cases of acute
PTE
a. Hill’s sign
a. 5%
b. Duoroziez’s sign
b. 10%
c. Traube’s sign
c. 20%
d. Beckers sign
d. 30%
267. Urokinase and streptokinase are contraindicated in
277. Preferred antihypertensive in patients of Benign Prostatic
a. Malignancy
Hypertrophy is
b. A-V fistula
a. ACE Inhibitors
c. Pulmonary embolism
b. Ca channel Blockers
d. Thrombophlebitis
c. Alpha Blockers
268. Loud S1 is seen in all except d. Beta Blockers
a. Short PR interval
278. DuctusVenosus is a low resistance bypass between
b. AF
a. Pulmonary vein & IVC
c. MS
b. Portal Vein & IVC
d. Rapid heart rate
c. Hepatic Vein & IVC
269. Reverse splitting of s2 is seen in d. None
a. RBBB
279. Fetal cardiac activity can be earliest detected by:
b. LBBB
a. 4week
c. LV Placed beats
b. 6 week
d. Lv ectopic beats
c. 5 week
d. 10 week
280. PulsusParadoxus is commonly seen in: 290. In pheochromocytoma there is increased level of 1137
a. Constrictive pericarditis a. Serum HMA
b. Cardiac tamponade b. Serum bradykinin
c. Restrictive cardiomyopathy c. Urinary VMA
d. Risperidone d. All of the above
281. Most common congenital heart disease in adults 291. An asymptomatic 44 year old male was detected with an
a. ASD abnormality on cardiac examination. His ECG showed a Left
Bundle Branch. His 2D Echo showed an Ejection fraction
b. Tetralogy of fallot of 55% with no regional wall motion abnormality and no
c. VSD significant valvular pathology. The abnormality detected on
d. bicuspid Aortic valve cardiac examination was:
a. Wide Splitting of the S1
282. 150/100 mm Hg falls in which category of Hypertension as per
b. Reverse Splitting of the S2
CARDIOLOGY
JNC7
c. Mid diastolic murmur at apex
a. stage-1
d. Continuous murmur in the infraclavicular area
b. Stage-II
c. stage-III 292. A 64 year old female was diagnosed with Severe Mitral
d. Stage of pre_Hypertension stenosis with moderate mitral regurgitation with LV Systolic
dysfunction. Her ECG showed sinus rhythm with a PR
283. What does the abbreviation CAVHD stand for? Interval of 220 msec. On cardiac examination her first heart
a. Coronary atrioventricular heart disease sound would be:
b. Cavernous atrioventricularhemopoetic disease a. Loud
c. Continuous Arterio Venous Hemo Dialysis b. Soft
d. Calcified aortic venous heterozygous disease c. Variable intensity
d. Wide Split
284. The left to right shunt is seen in all of the following except
a. PDA 293. A 30 years old female with Primary Pulmonary Hypertension
had a 2D Echo with normal LV and RV systolic function and
b. VSD an estimated PA Systolic pressure of 85 mmHg. This would
c. ASD be reflected on clinical examination as which of the following
d. TGA findings?
a. Widely Split S2 with Soft P2
285. Digoxin is used in which of the following conditions
b. Narrowly Split S2 with Soft P2
a. Atrial ectopics
c. Widely Split S2 with loud P2
b. Flutter with 2:1 block
d. Narrowly split S2 with Loud P2
c. Ventricular tachyarrhythmia
d. Ventricular bigeminy 294. An asymptomatic 11 year old male child was found to have
a wide and fixed split of the second heart sound on cardiac
286. The duration of the depolarisation of the heart from the SA examination. He had no murmurs and no other adventitious
node to the AV node is reflected on the ECG by sounds. His ECG and Chest X Ray were within normal limits.
What would be the next most appropriate investigation?
a. QT interval
a. Reassurance. No further investigations required.
b. PR interval
b. 2D Echocardiography
c. RS interval
c. Cardiac MRI
d. RR interval
d. ASO Titres
287. The most common cause of secondary hypertension in
children is 295. All the following sounds occur during diastolic phase of the
cardiac cycle EXCEPT
a. Renal artery stenosis
a. S4
b. Adrenal tumors
b. Opening Snap
c. Renal disease
c. Aortic Ejection Click
d. Coarctation of aorta
d. Pericardial Knock
288. Which are the values of Troponin T that are prognostically
significant? 296. Closure of the Mitral and Tricuspid Valve are following by
which phase of the cardiac cycle?
a. >/= to 0.05 ng/ml
a. Iso-volumetric Contraction
b. >/= to 0.1 ng/ml
b. Ejection
c. >/= to 0.001 ng/ml
c. Iso-volumetric Relaxation
d. >/= to 0.01 ng/ml
d. Diasthasis
289. Urokinase and streptokinase are contraindicated in
297. All the following regarding the Third Heart Sound are true
a. Malignancy
EXCEPT
b. A-V fistula
a. Occurs 120 – 200msec after the second heart sound
c. Pulmonary embolism
b. Occurs due to early rapid filling phase
d. Thrombophlebitis
1138 c. Can be heard in VSD and PDA 306. A 60 year old Diabetic and hypertensive male presented to
d. Occurs normally in elderly patients the casualty with severe chest pain radiating to the left arm
associated with severe breathlessness and orthopnoea. His
298. The Fourth Heart Sound can be heard in ECG showed an evolving anterior wall myocardial infarction.
All of the following finding on cardiac auscultation would
a. Presence of a Non-compliant left ventricle
indicate a complication of a Myocardial Infarction EXCEPT
b. Atrial fibrillation
a. S3-S4 gallop
c. Severe Mitral Stenosis
b. Pericardial Rub
d. Chronic Mitral Regurgitation
c. Early diastolic Murmur at the aortic area
299. Which Heart Sound corresponds with the ‘a’ wave of the JVP? d. Pan systolic murmur at the apex
a. S1
307. Which of the following findings would help to differentiate
b. S2 Hypertrophic Cardiomyopathy from Valvular Aortic Stenosis?
c. S3 a. Bisferians Pulse
d. S4 b. Opening Snap
MCQ's
300. A 55 year old lady with symptoms of exertional palpitations, c. Ejection Systolic Murmur with a delayed peak
insomnia, irritability and heat intolerance was diagnosed with d. S4
hyperthyroidism. The following would be expected findings
on cardiovascular system examination EXCEPT 308. All of the following findings would help to differentiate an
Austin Flint Murmur of Aortic Regurgitation from a Mid-
a. Atrial fibrillation
diastolic Murmur of Mitral Stenosis EXCEPT
b. Wide Pulse Pressure
a. Wide Pulse Pressure
c. Soft S1
b. S3
d. Means Lerman Scratch
c. JVP
301. A 55 year old male with history of Exertional angina and a d. Location of the Apex Impulse
single episode of syncope was found to have a Constant
Systolic Ejection Click at the Aortic Area, a soft A2, S4 and an 309. All the following indicate Severity of Mitral Regurgitation
Ejection Systolic Murmur with a delayed systolic peak. His EXCEPT
diagnosis is a. Soft S1
a. Bi-cuspid Aortic Valve with Severe Aortic Stenosis b. S3
b. Degenerative Calcified Severe Aortic Stenosis c. Mid Diastolic Murmur
c. Hypertrophic Cardiomyopathy with a Severe LV outflow d. Systolic Thrill
tract obstruction
d. Sub Valvular Membrane with severe aortic stenosis 310. A 26 year old male was admitted with Infective Endocarditis
with Severe Aortic Regurgitation. His cardiac auscultation
302. A 22 year old patient with Marfans syndrome can have the revealed all the following murmurs EXCEPT
following findings on cardiac examination EXCEPT a. Ejection systolic murmur at the aortic area radiating to the
a. Waterhammer Pulse Carotids
b. S4 b. Short Early Diastolic Murmur at the aortic area
c. Pulsations in the Suprasternal notch c. Mid Diastolic Murmur at the apex
d. Early diastolic murmur in aortic area d. Early Systolic murmur at the apex
303. A 46 year old male was diagnosed with Rheumatic Heart 311. A 26 year old male was diagnosed with Hypertrophic
disease with Mild Mitral Stenosis 8 years ago. He has gradual Cardiomyopathy. His cardiac examination revealed one of the
worsening of Dypsnea on exertion since 6 months. Which of following findings:
the following findings indicate increased severity of Mitral a. Pulsus Alternans
Stenosis
b. Aortic Ejection Click
a. Loud S1
c. Mid Systolic Murmur increasing in standing position
b. Intensity of Opening Snap
d. Mid Systolic Murmur decreasing in Phase II of Valsalvas
c. Wide A2-OS interval manouver.
d. Length of Mid Diastolic Murmur
312. The following combination of Murmur and Cardiac Pathology
304. All the following about Opening Snap are false EXCEPT is INCORRECT
a. Systolic sound occurring after S1 a. Gibsons Murmur – Patent Ductus Arteriosus
b. Indicates pliability of the mitral valve b. Rytands Murmur – Severe Aortic Regurgitation
c. A2 – OS interval correlates directly with severity of mitral c. Carey Coumbs murmur – Rheumatic Fever
stenosis d. Graham Steele’s Murmur – Hypertensive Pulmonary
d. Often heard in Calcific Mitral stenosis Regurgitation
305. The following sequence of diastolic sounds according to the 313. The following are causes of Continuous murmurs except:
timing of their occurrence is CORRECT: a. Aortopulmonary Window
a. A2-P2-OS-S3-S4 b. Rupture of Sinus of Valvsalva Aneurysm
b. P2-A2-OS-S3-S4 c. Post Bidirectional Glenns Shunt
c. A2-P2-S3-S4-OS d. Coronary AV Fistula
d. A2-P2-S3-OS-S4
314. A 7 year old boy presented with fatigue and reduced activity b. Left sided valvular heart disease 1139
since 1 year. On examination, he had pan-digital clubbing and c. Coronary artery disease with LV dysfunction
central cyanosis. He was diagnosed with Tetrology of Fallot.
d. ILD with pulmonary artery hypertension.
Which of the following findings would correlate with the
diagnosis of TOF ?
323. Peripheral edema may be seen in
a. Narrowly Split Second Heart Sound
a. Aortic stenosis
b. Loud P2
b. Constrictive pericarditis
c. Pan systolic murmur
c. Pericardial effusion
d. Continuous murmurs at the back and axilla
d. Atrial septal defect
315. Which of the following regarding response to Strain phase of
324. Reverse differential cyanosis may be seen in
Valvsalva Manouver is INCORRECT ?
a. PDA with PAH and reversal of shunt
a. HOCM Murmur – increases in intensity
b. Coarctation of aorta with interrupted aortic arch with PDA
b. Mitral Valve Prolapse – Click occurs later and Murmur is
CARDIOLOGY
having right to left shunt
louder
c. TGA with preductal narrowing, PAH and reverse flow
c. Aortic Stenosis Murmur – decreases in intensity
through PDA
d. Mitral Stenosis Murmur – decreases in intensity d. Double outlet right ventricle
316. The normal apical impulse is characterized by all except 325. Central cyanosis may be due to
a. Lowermost and outermost point of maximal impulse in late a. TOF with multiple aorto pulmonary collaterals
systole
b. Corrected transposition of great arteries
b. Located in 4th or 5th intercostal space inside the
c. Congestive heart failure
midclavicular line within 10 cm from midsternal line
d. Truncus arteriosus
c. Palpable area of 2-2.5 sq cm and localized to one intercostal
space 326. The closure of the aortic valve corresponds to which phase of
d. Gentle non sustained tap the pulse wave
a. Percussion wave
317. Left parasternal pulsations are palpable in all except
b. Anacrotic notch
a. Aortic regurgitation
c. Tidal wave
b. Tricuspid regurgitation
d. Dicrotic notch
c. Mitral regurgitation
d. Pulmonary stenosis 327. The pulse shown in the figure is characteristic of
330. A first year MBBS student has just learnt the art of BP 336. A 35 year old female is a known case of rheumatic heart disease
measurement. His professor will give him all the instructions with severe mitral stenosis. She has been complaining of
except progressively increase in breathlessness since the last 2 years.
a. Look for orthostatic hypotension in old age individuals Presently she has noticed symptoms of PND. The reason for
the PND may be all except
b. If thigh cuff is not available, arm cuff may be tied at the level
of ankle for measurement of lower limb BP a. Pulmonary arterial hypertension
c. The systolic pressure is estimated by the palpatory method b. Pulmonary venous hypertension
and diastolic pressure by the auscultatory method c. Decreased sympathetic drive during sleep
d. During BP measurement, the BP apparatus should be at the d. Increased venous return during supine position
MCQ's
333. The JVP is best timed with cardiac cycle by which of the 339. A 19 year old female presents with swelling of the feet. She
following has atrial fibrillation. Her JVP shows an attenuated X descent.
a. A wave is visible just before the carotid pulse What is her differential diagnosis
b. A wave coincides with S4 a. Severe mitral stenosis
c. X descent is between S1 and S2, X’ is simultaneous with b. Severe mitral stenosis with moderate mitral regurgitation
radial pulse c. Severe pulmonary regurgitation secondary to pulmonary
d. V wave peaks after S2 artery hypertension
d. Severe TR
334. The X’ descent is due to
a. Atrial relaxation 340. A prominent X and prominent Y descent may be seen in
b. Downward pull of tricuspid valve by contracting RV a. Constrictive pericarditis
c. Impact of carotid artery and upward bulging of closed b. Restrictive cardiomyopathy
tricuspid valve c. Dilated cardiomyopathy with heart failure
d. RA emptying when tricuspid valve opens d. HOCM
1141
Cardiology - Answers
1. d: This is a typical cause of stroke in a young person due to 28. d
prolonged immobilty. Deep vein thrombosis with patent
foramen ovale will cause paradoxical embolism and stroke. 29. b
2. c: Atrial myxomas are more often on the left. Though benign, 30. e
they can occlude the mitral valve and produce sudden
loss ofcardiac output. They may embolizesmallportions of 31. e
themselves or thrombus forned over their surface.
32. d
3. b: The uraemia leads to exudation of fibrin onto the epicardial and
pericardial surfaces. Haemorrhagic pericarditis is moretypical 33. a
CARDIOLOGY
of tuberculosis or metastatictumour. Serous pericarditis is more
34. e
typical of collagen vascular diseases.
35. e
4. d
36. c: This patient has NYHA stage II heart failure. Studies such as
5. e: The macrolides are associated with a prolongation of the QT
CIBISII and MERITHF reveal that betablockerssignificantlyreduce
interval. Other antimicrobials associated with prolonged QT
morbidity and mortality in heartfailure.
include quinine, levofloxacin.
37. d
6. e
38. d: Acanthocytes are seen in
7. b
abetalipoproteinaemia. Retinitis pigmentosa is seen in
8. c abetalipoproteinaemia. Mental retardation is not present but
motor abnormalities andneurodegenerative are seen.
9. a: There is a single sound in Fallot’s because of an absent P2.
Aortic stenosis leads to reversed splitting (also seen with LBBB 39. d: Quaternary syphilis involves the cardiovascular system
and ventricular pacemaker). In RBBB there is wide splitting of commonly in form of ascending aortic aneurysm and
S2 but it is not fixed. aorticregurgitation. Diphtheria,coxsackievirus,Chagas disease
and toxoplasmosis are all associated with myocarditis.
10. d: The suggestion here is that this man has coronary artery
disease with an impending myocardial infarction. Infarction of 40. b
the LAD would cause necrosis of the left ventricle. Thrombus
41. b
may form on an area of dyskinetic ventricle. Therefore he is
most at risk of embolus of thrombus from the LV.
42. d
11. b: Many drugs can cause a prolonged QT interval. more
43. c
12. c
44. c
13. e
45. e
14. a
46. c
15. b
47. e
16. a
48. b
17. a: The bioprosthesis has the advantage of not requiring
49. b: The results show normal T4, low T3 with elevated TSH. These
anticoagulation, but it does not wear well with time, and
results are typical of amiodarone induced hypothyroidismwhich
typically mustbe replaced within 5 to 10 years
inhibits the peripheral conversion of T4to T3.
18. b
50. e: This lady has Chronic Obstructive Airways disease and
subsequent CorPulmonale leading to right heart failure.
19. d
Nonbacterialthrombotic endocarditis is a conditionseen in frail
20. e ill individuals.
21. c 51. c
22. a 52. c
24. d 54. e
25. d 55. b
26. a 56. a
27. d 57. e
1142 58. d 85. e
59. a 86. e
60. b 87. a
61. e 88. c
62. a: Bicuspid aortic valve is perhaps the most common form 89. e
of congenital heart disease in adults. Bicuspid valves have
apropensity to wear out and calcify with aging.Bicuspid aortic 90. a: This man has alcoholic liver cirrhosis with ascites. The
valve tends to be a sporadic. cardiomyopathy of alcoholism is a dilated or congestive form.
63. c 91. a
64. c 92. b
65. b 93. c
MCQ's
66. b 94. b
67. b 95. d
68. e: The history is typical of aortic dissection. All the others 96. c
could cause sudden collapse but not with acute chest pain
radiatingto the back in the presence of arecent normal exercise 97. b
test. Acute MI is possible but not the most likely.
98. b
69. d: The Austin Flint murmur is a low frequency mid/late diastolic
murmur which may show presystolic accentuation which 99. e
isvirtually indistinguishable from that ofmitral stenosis. There
100. d
is no correlation between presence of murmur and severity of
AR, or aetiology. The first heart soundis normal but in severe
101. e
cases, it may beabsent.
102. a
70. d: Patients with HCM are at increased risk of sudden cardiac
death due to VF/VT. Implantable Cardio Defibrillators (ICD) 103. e
aresuperior to Amiodarone or BetaBlockers for preventing
this. Reducing outflow tract obstruction with myomectomy or 104. e
Alcohol Septal Ablation does notreduce the risk of SCD.
105. d: Selenium deficiency is one of the reversible causes of dilated
71. c cardiomyopathy.
72. e 106. e
73. d 107. e
74. b 108. b
CARDIOLOGY
127. e 167. a 207. d 247. a 287. c
• VSD
Loud S1 Soft S1 Variable S1
Normal in Mitral / Tricuspid Atrial fibrillation C. Others:-
children Regurgitation
• Normal in Children
Sinus tachycardia Mitral valve Complete heart
Prolapse block • Straight back Syndrome
Mitral Stenosis Calcified Mitral AV Dissociation
stenosis • Pectus Excavation
MCQ's
State
• TAPVC, Common Atruim
Short PR Interval Myocarditis
LBBB • Acute Pulmonary Embolism
LOUD T1 Thick chest wall, • Severe RV Failure
Obesity
Tricuspid stenosis Pericardial Reverse Split S2 may be due to delayed A2 or early P2
effusion
A. Delayed A2
RA Myxoma Emphysema
Esbteins anomaly • Electrical:-
o ASD
296. Cardiac Cycle 1145
Loud A2 Soft A2
Hyperkinetic Stales Age, Aortic Sclerosis
Systemic Hypertension Vascular Aortic stenosis
Aortic Aneurysm Vascular Aortic regurgitation
Aortic Root Pathology
(Syphilis)
Cong Bicuspid AV
Pulmonary Atresia
Loud P2 Soft P2
Hyperkinetic States Vascular Pulmonary stenosis
Pulmonary Hypertension Dysplastic Pulmonary Valve
CARDIOLOGY
ASD (even in absence of PH) Obesity, Thick Chest,
Emphysema
Thin Chest, Straight Back
Syndrome
• Inspiration leads to an increased venous return and no left • Co- insides with- Y Decent of JVP
to right shunt across the ASD.
Timing:
• Expiration leads to a reduced venous return and a increase
• PATHOLOGICAL: 140-160msec after A2
in the left to right shunt across the ASD. Therefore there is
an increased RV filling in both expiration &inspiration that
• PHYSIOLOGICAL: 20-200 msec after A2
produces a fixed split.
Causes of S3
B. Increased Pulmonary Capacitance due to ADS leads to
an increased pulmonary blood flow. There is no further • PHYSIOLOGICAL:
increase in pulmonary blood flow during inspiration
leading to a fixed split. o Children
A2- P2- Opening Snap- Pericardial Knock – Tumour Plop - S3- o Anxiety
S4
o Exercise
Timing of Diastolic Sounds in Relation to the Second Heart
Sound: • PATHOLOGICAL:
o Hyperdynamic precordium, wide pulse pressure The A2-OS interval is inversely proportionate to the severity of
mitral stenosis. (Refer to explanation in Q14)
o Atrial fibrillation
The length of the mid diastolic murmur indicates the duration
o Cardiac Hypertrophy of time for which the gradient across the mitral valve is
maintained.
o Systolic Hypertension
As the severity increases, the longer is the gradient sustained
o Congestive cardiac failure and the longer is the mid-diastolic murmur.
The Means–Lerman scratch is arare murmur found in patients 304. OPENING SNAP
with hyperthyroidism. It is a mid-systolic scratching sound
best heard over the upper part of the sternum or second left Characteristics
intercostal space at the end of expiration.
• High Frequency Sounds
301. Aortic Click
• Early Diastolic Sound - A2-OS= 30-150 msec
Characteristics
• Indicates pliable mitral valve
• S1-EC= 50 msec
• Intensityof OS parallels Loudness of S1
• Coincides with ANACROTIC NOTCH on the upstroke of
aortic pressure Trace Pre-Requisite:
• Constant click: No respiratory variation • Velocity across AV Valves High- Rapid excursion of leaflets
Causes: 1147
Austin Flint Mid-Diastolic
Murmur of Aortic Murmur of Mitral
• Mitral Stenosis
Regurgitation Stenosis
• Rare Causes 6. S1 Normal / Loud
Decreased
o Tricuspid stenosis
7. S3 Present Absent
o MR, TR 8. Opening Snap Absent Present
o VSD,PDA, ASD, Ebsteins 9. Diastolic Thrill Absent Present
10. Presystolic Absent Present
o HOCM Accentuation
of MDM
A2-OS Interval: As the severity of mitral stenosis increases, the
opening snap occurs earlier in diastole. As the severity of MS 11. Exercise / Decreases Increases murmur
CARDIOLOGY
increases, the LA pressure is higher and thus the earlier does the Amyl Nitrate murmur
LA pressure increase above the LV diastolic pressure to produce
opening of the mitral valve and the opening snap. 309. Severity of Mitral Regurgitation
Therefore, the A2-OS interval is inversely proportionate to the Indicators of Severity of MR are :
severity of the mitral stenosis.
• Presence of S3 at the apex that indicates increased flow
Absent OS in MS: across the MV due to increased regurgitant volume.
305. Diastolic Heart Sounds: Refer to explanation of Q5 Aortic Regurgitation (AR) produces a Early Diastolic Murmur
(EDM) at the aortic area or in the left third of fourth parasternal
306. Auscultation during Acute Myocardial Infarction area. The length of the EDM correlates directly with the severity
of AR (more severe AR produces longer diastolic murmur).
During an acute myocardial infarction, auscultation can assist in An exception is in acute AR where the murmur may be short
determining certain complications. in spite of severe AR due to high left ventricular end-diastolic
pressure.
a. A S3-S4 gallop indicates Congestive cardiac failure.
Due to the increased regurgitant volume, an Ejection Systolic
b. A Pericardial Rub indicates Pericarditis Murmur at the aortic area can be heard radiating to the Carotids
(flow murmur). Thus radiation to the carotids is not an indicator
c. Pan systolic murmur at the apex indicates a mitral of organic aortic stenosis.
regurgitation due to papillary muscle dysfunction or
rupture. A pan systolic murmur in the lower parasternal Severe ‘free’ aortic regurgitation can lead to production of a Mid-
area indicates a post MI ventricular septal defect. Diastolic Murmur at the apex called the Austin Flint Murmur. It is
produced as the regurgitant jet strikes the anterior leaflet of the
307. Bisferians Pulse in Hypertrophic Cardiomyopathy mitral valve and leads to vibration of the leaflet producing an
MDM. Other theories of production of the Austin flint murmur
The type of pulse felt in a Hypertrophic cardiomyopathy is
are premature closure of MV due to AR or raised LVEDP in AR
a bisferians pulse (twice peaking pulse) with both peaks in
producing a diastolic mitral regurgitation producing a MDM.
systole. The type of pulse felt in valvular aortic stenosis is Pulsus
parvus et tardus. 311. Dynamic Auscultation in Hypertrophic Cardiomyopathy
308. Differentiating Austin Flint Murmur of Aortic The LVOT gradient in HOCM can be affected by changes in the
Regurgitationfrom a Mid-Diastolic Murmur of Mitral Stenosis myocardial contractility, ventricular volume or arterial pressure.
• Absent P2
Increases in Intensity Decreases in Intensity
Valsalva manouver (phase II) Isometric hand grip • Aortic Vascular Ejection Click (due to dilated aorta)
Standing (from supine Supine (from standing • Short Ejection systolic murmur (inversely related to the
position) position) severity of TOF)
Amyl nitrate Squatting
• Continuous murmurs of bronchopulmonary collaterals in
During or after Exercise Beta blockers the axilla, back or chest
After a Ventricular
premature beat • Continuous murmur in patient post – Blalock Taussig Shunt
c. Renal, Carotid or Mesentric Artery Stenosis Pressure overload - PAH, Moderate to severe MR
Primary PAH, PS
C. Increased Flow Volume overload - Moderate Regional wall motion
TR, ASD, VSD abnormality of LV
a. Venous Hum
b. Mammary Souffle
318. c: There is increased amplitude and duration of LV ejection 1149
Cardiovascular Pulmonary Gastrointestinal
The apex beat is examined in supine position and trunk elevated Chronic fibrosing
to 30 degree for localization. The character of apex beat is alveolitis
examined in the left lateral position.
Tapping apex beat - It is characterized by a sharp, short tap due Differential Unilateral Unidigital
to reduced filling of LV. It is equivalent of loud S1. clubbing clubbing clubbing
PDA with shunt Aortic aneurysm Hereditary
Hyperdynamic apex - There is an increase in amplitude and
reversal
duration of excursion of apical impulse. It gives a partial lift to
the examining fingers. It is ill sustained, <50% of systole. It is Brachial Median n injury
seen in volume overload states and eccentric LVH. arteriovenous
fistula
Heaving apex - There is an increased amplitude and duration of
Pancoast tumor Tophaceous gout
excursion of the apical impulse. This is due to increased duration
CARDIOLOGY
of LV ejection. There is a sustained lift of the examining fingers Erythromelalgia Sarcoidosis
which lasts >50% of systole. It is seen in pressure overload Lymphangitis Trauma
conditions and concentric LVH.
322. d: ILD with pulmonary artery hypertension.
Types of Apex beat
Progressively increasing breathlessness with episodes of PND
Tapping Hyperdynamic Heaving suggests a cardiovascular origin of breathlessness.
MS AR, MR Pressure overload - 323. b: Constrictive pericarditis
AS, HCM, systemic
hypertension Causes of peripheral edema
PDA, VSD Severe LV dysfunction
AV fistula LV aneurysm Bilateral Unilateral Bilateral non Unilateral
pitting pitting pitting non pitting
Blalock and Waterson Severe AR
shunt CHF Trauma Myxedema Lymphatic
obstruction
Anemia, Pregnancy, Severe ischemic MR - filariasis,
Thyrotoxicosis radiation,
Thin chest, pectus trauma, ma-
excavatum lignancy
Constrictive Inflamma- Milroy’s
319. d: Chronic severe mitral regurgitation pericarditis tion disease
Renal - Baker’s cyst
Palpable S3 Palpable S4
Glomeru-
Left ventricular failure, DCM AS lonephritis,
Nephrotic
Chronic severe MR Hypertrophic
syndrome
Cardiomyopathy (HCM)
Cirrhosis of Varicose
Children Acute MR, Acute AR
liver veins
Pregnancy CAD
Protein los- DVT
320. c: Massive pericardial effusion ing enteropa-
thy
Rotch’s sign - In massive pericardial effusion, the cardio hepatic Nutritional Congenital
border becomes obtuse. There may be associated dullness in - anemia, venous mal-
2nd ICS also. hypoprotein- formation
emia
321. d: ASD
Pregnancy,
Causes of clubbing pre menstru-
al edema
Cardiovascular Pulmonary Gastrointestinal
324. c: TGA with preductal narrowing, PAH and reverse flow
Cyanotic CHD Bronchiectasis Crohn’s disease through PDA
- TOF, TGA,
TAPVC, Tricuspid Differential cyanosis - Cyanosis of lower limbs with no cyanosis
Atresia of upper limbs
Infective Emphysema Ulcerative colitis i. PDA with PAH and reversal of shunt
Endocarditis
Myxoid tumor Empyema Biliary cirrhosis ii. Coarctation of aorta with interrupted aortic arch with PDA
having right to left shunt - Oxygenated blood goes to upper
Lung abscess Polyposis of colon parts of the body and desaturated blood goes to lower part
Bronchogenic of the body via the PDA
carcinoma
Pulmonary
tuberculosis
1150 Reverse differential cyanosis - Cyanosis of fingers which exceeds a. Hyperkinetic (Bounding) Pulse - It has larger pulse wave
that of toes amplitude. It is due to increase in left ventricular ejection,
stroke volume, arterial pressure, sympathetic activity or
i. TGA with preductal narrowing (coarctation or interrupted decreased arterial compliance. It is seen in i) elderly subjects
aortic arch), PAH and reverse flow through PDA with arteriosclerosis and systolic hypertension, ii) anxiety,
iii) anemia, iv) thyrotoxicosis, v) exercise, vi) hot and humid
ii. DORV with subpulmonary VSD, PAH and reverse flow environment, vii) alcohol intake and viii) high output states
through PDA with increased distal arterial runoff like aortic regurgitation,
patent ductus arteriosus, large A-V fistula, Paget’s disease
iii. TGA with intact ventricular septum, PAH and reverse flow and severe cirrhosis.
through PDA
b. Hypokinetic Pulse - A small or diminished pulse is due
325. d: Truncus arteriosus to low cardiac output with reduced left ventricular stroke
volume, shorter left ventricular ejection time or intense
Causes of Cyanosis
vasoconstriction. An unsustained pulse suggests decreased
stroke volume without left ventricular outflow obstruction,
Central cyanosis Peripheral Mixed cyanosis whereas a slow rising sustained pulse of small volume
MCQ's
iii. Dicrotic pulse - The first wave occurs in systole and the
second accentuated component is a diastolic reflection
wave occurring in diastole. On simultaneous auscultation,
S2 separates the two pulse waves. It is better appreciated
during inspiration. It is seen in young patients having
cardiomyopathy with severe left ventricular dysfunction,
low cardiac output, low blood pressure, high systemic
vascular resistance, tachycardia, during inspiration in
pericardial tamponade, post valve replacement for aortic or
mitral regurgitation with left ventricular dysfunction and
occasionally with fever in young.
Figure 3
328. b: Dilated cardiomyopathy with severe left ventricular when the radial pulse is no longer palpable. This palpatory 1151
dysfunction method prevents underestimation of blood pressure due to
auscultatory gap. The systolic and diastolic pressures are then
Pulsus alternans - It is present during sinus rhythm when estimated by auscultatory method. Cuff should be deflated at
patient’s peak systolic arterial pressure and pulse volume <3 mm Hg/sec. The column should be read to the nearest 2 mm
are alternately strong and weak. It occurs due to beat to beat Hg. The level of peak systolic pressure is the point at which two
alteration in left ventricular ejection pressure and signifies consecutive Korotkoff sounds are heard. The disappearance of
severe left ventricular dysfunction. (It is not related to electrical the Korotkoff sounds is the true diastolic pressure. After every
alternans which has a beat to beat variation in the amplitude of cuff inflation, deflate the cuff completely and allow sufficient
QRS complex as seen in massive pericardial effusions). It is best time for venous return. There should be no talking between
appreciated clinically in the radial or brachial arteries. It may be the subject and observer. In severe aortic regurgitation and
associated with signs of heart failure like S3 gallop. It can also hyperkinetic circulatory states, the diastolic pressure should
be detected by slow decompression of the sphygmomanometer be recorded in both phase IV and V. In atrial fibrillation, there
cuff while listening to the alteration of Korotkoff sounds. When is beat to beat variation in blood pressure; hence an average of
systolic pressure alternates by >20 mm Hg it can be detected three readings is taken as a blood pressure. For measurement
by palpation of the peripheral pulse with patient’s breath
CARDIOLOGY
of lower limb pressure, the patient lies prone; the thigh cuff is
held in deep expiration. It is accentuated after a PVC, Valsalva wrapped around and auscultate the popliteal fossa. If a thigh
maneuver, abrupt upright posture or deep inspiration. cuff is not available, an arm cuff can be wrapped around the
lower leg and auscultate the posterior tibial artery or dorsalis
329. b: Pericardial tamponade pedis artery with a pediatric bell chest piece.
Pulsus paradoxus - This term was coined by Kussmaul. There 331. b: Mean BP=Diastolic BP + 1/3 pulse pressure
is marked and exaggerated inspiratory fall in systolic blood
pressure in which palpable peripheral arterial pulse and audible Cardiac output is the product of heart rate and stroke volume
Korotkoff sounds disappear in inspiration. The blood pressure (CO=HRxSV). Arterial blood pressure is the product of cardiac
cuff is inflated beyond the peak systolic pressure and slowly output and peripheral resistance (BP=COxPR). The pulse
deflated. The degree of paradoxus is the difference between the pressure is the difference between systolic and diastolic blood
systolic pressure at which the Korotkoff sounds are first heard pressure (approximately 40 mm Hg). Mean blood pressure is
during expiration and the point at which all beats are well the sum of diastolic blood pressure and 1/3rd pulse pressure
heard during both phases of respiration. The word paradoxus (approximately 95-100 mm Hg).
is a misnomer because systolic pressure normally falls during
inspiration by 4-6 mm Hg. In pulsus paradoxus the difference 332. d: Any position where the peak of the venous column is well
is >10 mm Hg. The patient must be breathing quietly and not identified
deep breathing or performing Valsalva maneuver. It is seen in
pericardial tamponade, constrictive pericarditis, emphysema, The patient should be reclining comfortably without any
asthma, severe congestive cardiac failure and marked obesity. tension on neck tissues. The chin is elevated and head rotated
to the left. It is preferable to have tangential lighting. Lean over
Mechanism - Cardiac tamponade is a continuum from effusion to the left side of the patient while examining the right side of
to full blown circulatory collapse. The hemodynamic effects the neck. The sternal angle of Louis is 5 cm above the mid right
depend on the amount of effusion and the pericardial pressure atrium whether supine, 45 degree or 90 degree position is given.
volume relationship. As fluid accumulates in pericardial sac, The venous pressure is measured from the angle of Louis. The
there is increased left and right sided atrial and ventricular thorax should be positioned at an angle where the peak of the
pressures which equalize at a pressure similar to intrapericardial venous column is well identified. If the venous pressure is too
pressure. Inspiration increases the venous return to the right side low, place the patient supine with leg elevation and ask him to
of the heart at a period when the total heart volume is fixed. This take deep breaths. If the pressure is too high, the pulsations may
increases the right ventricular diastolic dimensions pushing the be behind the angle of the mandible, so keep the patient at 90
interventricular septum to the left. This reduces left ventricular degree and examine. The height of the A and V wave during
dimensions, compliance and filling. Also inspiratory pooling inspiration is taken as the venous pressure. Two scale method
of blood into the pulmonary circulation causes under filling of is used. A horizontal scale at the peak of the venous column
the left atrium and left ventricle. The under filled left ventricle cuts the vertical scale kept at the angle of Louis. For supine 2
in tamponade operates on the steep ascending limb of Starling cm is the upper limit of normal and for 45 degree 4.5 cm is the
curve, so inspiratory reduction of left ventricle filling causes a upper limit of normal for venous pulsations. By adding 5 cm, we
marked depression of stroke volume and systolic pressure. can obtain the actual venous pressure. If it is not visible, deep
inspiration can bring out the waves. The normal level is 4 cm
Reverse pulsus paradoxus may be seen in cardiac tamponade above the angle of Louis which is equal to 9 cm of water or 6 mm
with positive pressure ventilation and isorhythmic AV Hg.
dissociation.
333. c: X descent is between S1 and S2, X’ is simultaneous with radial
Pulsus paradoxus may be absent in cases of cardiac tamponade pulse
with atrial septal defect/ ventricular septal defect/ aortic
regurgitation/ pericardial adhesions. It is recommended to use the X and Y descent to time the venous
pulse. The negative X descent is between S1 and S2 and X’
330. c: The systolic pressure is estimated by the palpatory method is simultaneous with radial pulse. The A wave is visible as a
and diastolic pressure by the auscultatory method flickering pulsation just before the carotid pulse is felt. During
auscultation, the A wave coincides with S4 and is almost
Measurement of blood pressure - The blood pressure is simultaneous with S1. The V wave peaks just after S2 and Y
measured with an aneroid or mercury manometer. The bladder descent begins after the V wave.
length/width should be 80%/40% of the arm circumference.
The ratio of cuff width to length should be 1:2. The center of 334. b: Downward pull of tricuspid valve by contracting RV
the rubber bladder should be on the brachial artery. The blood
pressure cuff is wrapped tightly around the arm. The diaphragm Careful examination of the venous pulse can provide useful
or bell of the stethoscope is firmly placed at the brachial artery, information about the right sided cardiac physiology (Figure
so that the upper edge of the stethoscope is in contact with the 4). When the right atrium contracts, its pressure rises pushing
distal edge of the cuff. The patient should be seated comfortably, the blood from right atrium to the right ventricle at end of
back supported, bared upper arm, legs uncrossed with the arm ventricular diastole. It also causes blood to flow retrogradely
at level of heart. The cuff is inflated 20 mm Hg above the point into the superior vena cava and jugular veins. This produces a
1152 positive wave called the A wave. The A wave begins at the peak
A waves X descent V waves Y descent
of the P wave of ECG, immediately prior to S1 and the carotid
upstroke. As the right atrium starts relaxing, pressure falls Junctional TR Hypovole- TS, RA
causing the early portion of X descent. Simultaneously, the right rhythm, VT, mia myxoma
ventricular systole commences causing the intraventricular Isorhythmic
pressure to rise above the atrial pressure. This leads to closure AV dissocia-
of the tricuspid valve. The upward bulging motion of closed tion
tricuspid valve during isovolumic systole produces a positive CHB, Use of ni- Pericardial
wave called C wave. The onset of C wave corresponds to the Ventricular trates tamponade
tricuspid component of S1. It can be confused with transmitted pacing
carotid pulsations. It is usually not visible as a separate wave.
As the right atrial relaxation continues, right atrial pressure Absent A
falls during early right ventricular systole. During this phase waves
the tricuspid valve ring is also pulled downwards. The latter AF, Sinus
part of X descent, X’ reaches its lowest point. The X descent tachycardia
begins during systole and ends before S2. The great veins empty
into the right atrium during ventricular systole with a closed 336. a: Pulmonary arterial hypertension
MCQ's
CARDIOLOGY
vi. Becker’s sign - Prominent retinal artery pulsations
Upper limb
xiv. Palfrey’s sign - Pistol shot sounds over the radial artery
Figure2
Lower limb
Abdomen
339. d: Severe TR
Figure 4